Chapter 1 Mathemtical Introduction
1.1 Linear Vector Spaces: Basics
Exercise 1.1.1 Verify these claims. For the first consider ∣ 0 ⟩ + ∣ 0 ′ ⟩ |0\rangle+|0^{\prime}\rangle ∣0 ⟩ + ∣ 0 ′ ⟩ and use the advertised properties of the two null vectors in turn. For the second start with ∣ 0 ⟩ = ( 0 + 1 ) ∣ V ⟩ + ∣ − V ⟩ |0\rangle=(0+1)|V\rangle+|-V\rangle ∣0 ⟩ = ( 0 + 1 ) ∣ V ⟩ + ∣ − V ⟩ . For the third, begin with ∣ V ⟩ + ( − ∣ V ⟩ ) = 0 ∣ V ⟩ = ∣ 0 ⟩ |V\rangle+(-|V\rangle)=0|V\rangle=|0\rangle ∣ V ⟩ + ( − ∣ V ⟩) = 0∣ V ⟩ = ∣0 ⟩ . For the last, let ∣ W ⟩ |W\rangle ∣ W ⟩ also satisfy ∣ V ⟩ + ∣ W ⟩ = ∣ 0 ⟩ |V\rangle+|W\rangle=|0\rangle ∣ V ⟩ + ∣ W ⟩ = ∣0 ⟩ . Since ∣ 0 ⟩ |0\rangle ∣0 ⟩ is unique, this means ∣ V ⟩ + ∣ W ⟩ = ∣ V ⟩ + ∣ − V ⟩ |V\rangle+|W\rangle=|V\rangle+|-V\rangle ∣ V ⟩ + ∣ W ⟩ = ∣ V ⟩ + ∣ − V ⟩ . Take it from here.
Solution.
(1) ∣ 0 ⟩ |0\rangle ∣0 ⟩ is unique.
Proof. For an arbitrary state ket ∣ V ⟩ |V\rangle ∣ V ⟩ ,
(a) ∣ V ⟩ + ∣ 0 ⟩ = ∣ V ⟩ |V\rangle+|0\rangle=|V\rangle ∣ V ⟩ + ∣0 ⟩ = ∣ V ⟩
(b) ∣ V ⟩ + ∣ 0 ′ ⟩ = ∣ V ⟩ |V\rangle+|0^{\prime}\rangle=|V\rangle ∣ V ⟩ + ∣ 0 ′ ⟩ = ∣ V ⟩
Set ∣ V ⟩ = ∣ 0 ′ ⟩ |V\rangle=|0^{\prime}\rangle ∣ V ⟩ = ∣ 0 ′ ⟩ in (a) ⇒ \Rightarrow ⇒ ∣ 0 ′ ⟩ + ∣ 0 ⟩ = ∣ 0 ′ ⟩ |0^{\prime}\rangle+|0\rangle=|0^{\prime}\rangle ∣ 0 ′ ⟩ + ∣0 ⟩ = ∣ 0 ′ ⟩ ;
Set ∣ V ⟩ = ∣ 0 ⟩ |V\rangle=|0\rangle ∣ V ⟩ = ∣0 ⟩ in (ii) ⇒ \Rightarrow ⇒ ∣ 0 ⟩ + ∣ 0 ′ ⟩ = ∣ 0 ⟩ |0\rangle+|0^{\prime}\rangle=|0\rangle ∣0 ⟩ + ∣ 0 ′ ⟩ = ∣0 ⟩ ;
Therefore, by commutativity of vector addition, we have
∣ 0 ′ ⟩ = ∣ 0 ′ ⟩ + ∣ 0 ⟩ = ∣ 0 ⟩ + ∣ 0 ′ ⟩ = ∣ 0 ⟩ |0^{\prime}\rangle=|0^{\prime}\rangle+|0\rangle=|0\rangle+|0^{\prime}\rangle=|0\rangle ∣ 0 ′ ⟩ = ∣ 0 ′ ⟩ + ∣0 ⟩ = ∣0 ⟩ + ∣ 0 ′ ⟩ = ∣0 ⟩
(2) 0 ∣ V ⟩ = ∣ 0 ⟩ 0|V\rangle=|0\rangle 0∣ V ⟩ = ∣0 ⟩
Proof. We have
1 ∣ V ⟩ = ( 1 + 0 ) ∣ V ⟩ = 1 ∣ V ⟩ + 0 ∣ V ⟩ 1|V\rangle=(1+0)|V\rangle=1|V\rangle+0|V\rangle 1∣ V ⟩ = ( 1 + 0 ) ∣ V ⟩ = 1∣ V ⟩ + 0∣ V ⟩
where 1 ∣ V ⟩ = ∣ V ⟩ 1|V\rangle=|V\rangle 1∣ V ⟩ = ∣ V ⟩ . Therefore
∣ V ⟩ = ∣ V ⟩ + 0 ∣ V ⟩ |V\rangle=|V\rangle+0|V\rangle ∣ V ⟩ = ∣ V ⟩ + 0∣ V ⟩
Since ∣ V ⟩ |V\rangle ∣ V ⟩ is arbitrary here, compared with the definition of ∣ 0 ⟩ |0\rangle ∣0 ⟩ , which is ∣ V ⟩ + ∣ 0 ⟩ = ∣ V ⟩ |V\rangle+|0\rangle=|V\rangle ∣ V ⟩ + ∣0 ⟩ = ∣ V ⟩ for any ∣ V ⟩ |V\rangle ∣ V ⟩ , plus ∣ 0 ⟩ |0\rangle ∣0 ⟩ is unique, we can conclude that
0 ∣ V ⟩ = ∣ 0 ⟩ . 0|V\rangle=|0\rangle. 0∣ V ⟩ = ∣0 ⟩ .
(3) ∣ − V ⟩ = − ∣ V ⟩ |-V\rangle=-|V\rangle ∣ − V ⟩ = − ∣ V ⟩
Proof. For arbitrary ∣ V ⟩ |V\rangle ∣ V ⟩ ,
∣ V ⟩ + ( − ∣ V ⟩ ) = 0 ∣ V ⟩ = ∣ 0 ⟩ . |V\rangle+(-|V\rangle)=0|V\rangle=|0\rangle. ∣ V ⟩ + ( − ∣ V ⟩) = 0∣ V ⟩ = ∣0 ⟩ .
By definition of ∣ − V ⟩ |-V\rangle ∣ − V ⟩ , which is ∣ V ⟩ + ∣ − V ⟩ = 0 |V\rangle+|-V\rangle=0 ∣ V ⟩ + ∣ − V ⟩ = 0 , we can conclude that
∣ − V ⟩ = − ∣ V ⟩ . |-V\rangle=-|V\rangle. ∣ − V ⟩ = − ∣ V ⟩ .
(4) ∣ − V ⟩ |-V\rangle ∣ − V ⟩ is the unique addtive inverse of ∣ V ⟩ |V\rangle ∣ V ⟩ .
Proof. Suppose there exists another vector ∣ W ⟩ |W\rangle ∣ W ⟩ , satisfying ∣ W ⟩ = − ∣ V ⟩ |W\rangle=-|V\rangle ∣ W ⟩ = − ∣ V ⟩ , then
∣ V ⟩ + ∣ W ⟩ = ∣ V ⟩ − ∣ V ⟩ = ( 1 − 1 ) ∣ V ⟩ = 0 ∣ V ⟩ = ∣ 0 ⟩ \begin{aligned}
|V\rangle+|W\rangle&=|V\rangle-|V\rangle\\
&=(1-1)|V\rangle\\
&=0|V\rangle\\
&=|0\rangle
\end{aligned} ∣ V ⟩ + ∣ W ⟩ = ∣ V ⟩ − ∣ V ⟩ = ( 1 − 1 ) ∣ V ⟩ = 0∣ V ⟩ = ∣0 ⟩
Add ∣ − V ⟩ |-V\rangle ∣ − V ⟩ on the both sides, we have
∣ V ⟩ + ∣ W ⟩ + ∣ − V ⟩ = ∣ 0 ⟩ + ∣ − V ⟩ ∣ W ⟩ + ( ∣ V ⟩ + ∣ − V ⟩ ) = ∣ − V ⟩ ∣ W ⟩ + ∣ 0 ⟩ = ∣ − V ⟩ \begin{aligned}
|V\rangle+|W\rangle+|-V\rangle&=|0\rangle+|-V\rangle\\
|W\rangle+(|V\rangle+|-V\rangle)&=|-V\rangle\\
|W\rangle+|0\rangle&=|-V\rangle
\end{aligned} ∣ V ⟩ + ∣ W ⟩ + ∣ − V ⟩ ∣ W ⟩ + ( ∣ V ⟩ + ∣ − V ⟩) ∣ W ⟩ + ∣0 ⟩ = ∣0 ⟩ + ∣ − V ⟩ = ∣ − V ⟩ = ∣ − V ⟩
Therefore,
∣ W ⟩ = ∣ − V ⟩ |W\rangle=|-V\rangle ∣ W ⟩ = ∣ − V ⟩
■ ~\tag*{$\blacksquare$} ■
Exercise 1.1.2 Consider the set of all entities of the form ( a , b , c ) (a, b, c) ( a , b , c ) where the entries are real numbers. Addition and scalar multiplication are defined as follows:
( a , b , c ) + ( d , e , f ) = ( a + d , b + e , c + f ) α ( a , b , c ) = ( α a , α b , α c ) . \begin{aligned}
(a, b, c)+(d, e, f)&=(a+d, b+e, c+f) \\
\alpha(a, b, c)&=(\alpha a, \alpha b, \alpha c).
\end{aligned} ( a , b , c ) + ( d , e , f ) α ( a , b , c ) = ( a + d , b + e , c + f ) = ( α a , α b , α c ) .
Write down the null vector and inverse of ( a , b , c ) (a, b, c) ( a , b , c ) . Show that vectors of the form ( a , b , 1 ) (a, b, 1) ( a , b , 1 ) do not form a vector space.
Solution.
Null vector of ( a , b , c ) (a,b,c) ( a , b , c ) : By definition, for any ∣ V ⟩ |V\rangle ∣ V ⟩ ,
∣ V ⟩ + ∣ 0 ⟩ = ∣ V ⟩ . |V\rangle+|0\rangle=|V\rangle. ∣ V ⟩ + ∣0 ⟩ = ∣ V ⟩ .
Set ∣ 0 ⟩ = ( a 0 , b 0 , c 0 ) |0\rangle=(a_{0},b_{0},c_{0}) ∣0 ⟩ = ( a 0 , b 0 , c 0 ) , ∣ V ⟩ = ( a , b , c ) |V\rangle=(a,b,c) ∣ V ⟩ = ( a , b , c ) , where a , b , c a,b,c a , b , c are arbitrary numbers. Then
∣ 0 ⟩ + ∣ V ⟩ = ( a 0 , b 0 , c 0 ) + ( a , b , c ) = ( a 0 + a , b 0 + b , c 0 + c ) = ∣ V ⟩ = ( a , b , c ) \begin{aligned}
|0\rangle+|V\rangle&=(a_{0},b_{0},c_{0})+(a,b,c)\\
&=(a_{0}+a,b_{0}+b,c_{0}+c)\\
&=|V\rangle\\
&=(a,b,c)
\end{aligned} ∣0 ⟩ + ∣ V ⟩ = ( a 0 , b 0 , c 0 ) + ( a , b , c ) = ( a 0 + a , b 0 + b , c 0 + c ) = ∣ V ⟩ = ( a , b , c )
⇒ { a 0 + a = a b 0 + b = b c 0 + c = c ⇒ { a 0 = 0 b 0 = 0 c 0 = 0 \Rightarrow\quad\left\{
\begin{aligned}
a_{0}+a=a\\
b_{0}+b=b\\
c_{0}+c=c
\end{aligned}
\right.\quad \Rightarrow\quad\left\{
\begin{aligned}
a_{0}=0\\
b_{0}=0\\
c_{0}=0
\end{aligned}
\right. ⇒ ⎩ ⎨ ⎧ a 0 + a = a b 0 + b = b c 0 + c = c ⇒ ⎩ ⎨ ⎧ a 0 = 0 b 0 = 0 c 0 = 0
Therefore, the null vector of ( a , b , c ) (a,b,c) ( a , b , c ) is ( 0 , 0 , 0 ) (0,0,0) ( 0 , 0 , 0 ) .
Inverse vector of ( a , b , c ) (a,b,c) ( a , b , c ) : Suppose the inverse vector of ( a , b , c ) (a,b,c) ( a , b , c ) is ( a ˉ , b ˉ , c ˉ ) (\bar{a},\bar{b},\bar{c}) ( a ˉ , b ˉ , c ˉ ) . By definition,
( a , b , c ) + ( a ˉ , b ˉ , c ˉ ) = ∣ 0 ⟩ = ( 0 , 0 , 0 ) (a,b,c)+(\bar{a},\bar{b},\bar{c})=|0\rangle=(0,0,0) ( a , b , c ) + ( a ˉ , b ˉ , c ˉ ) = ∣0 ⟩ = ( 0 , 0 , 0 )
( a + a ˉ , b + b ˉ , c + c ˉ ) = ( 0 , 0 , 0 ) (a+\bar{a},b+\bar{b},c+\bar{c})=(0,0,0) ( a + a ˉ , b + b ˉ , c + c ˉ ) = ( 0 , 0 , 0 )
⇒ { a + a ˉ = 0 b + b ˉ = 0 c + c ˉ = 0 ⇒ { a ˉ = − a b ˉ = − b c ˉ = − c \Rightarrow\quad\left\{
\begin{aligned}
a+\bar{a}=0\\
b+\bar{b}=0\\
c+\bar{c}=0
\end{aligned}
\right.\quad \Rightarrow\quad\left\{
\begin{aligned}
\bar{a}=-a\\
\bar{b}=-b\\
\bar{c}=-c
\end{aligned}
\right. ⇒ ⎩ ⎨ ⎧ a + a ˉ = 0 b + b ˉ = 0 c + c ˉ = 0 ⇒ ⎩ ⎨ ⎧ a ˉ = − a b ˉ = − b c ˉ = − c
Therefore, the inverse vector of ( a , b , c ) (a,b,c) ( a , b , c ) is ( − a , − b , − c ) (-a,-b,-c) ( − a , − b , − c ) .
{ ( a , b , 1 ) } \{(a,b,1)\} {( a , b , 1 )} does not form a vector space since
(a) It violates the closure under addition, i.e.
( a 1 , b 1 , 1 ) + ( a 2 , b 2 , 1 ) = ( a 1 + a 2 , b 1 + b 2 , 2 ) ∉ { ( a , b , 1 ) } . (a_{1},b_{1},1)+(a_{2},b_{2},1)=(a_{1}+a_{2},b_{1}+b_{2},2)\notin \{(a,b,1)\}. ( a 1 , b 1 , 1 ) + ( a 2 , b 2 , 1 ) = ( a 1 + a 2 , b 1 + b 2 , 2 ) ∈ / {( a , b , 1 )} .
(b) It violates the closure under scalar multiplication, i.e.
ω ( a 1 , b 1 , 1 ) = ( ω a 1 , ω b 1 , ω ) ∉ { ( a , b , 1 ) } \omega(a_{1},b_{1},1)=(\omega a_{1},\omega b_{1},\omega)\notin\{(a,b,1)\} ω ( a 1 , b 1 , 1 ) = ( ω a 1 , ω b 1 , ω ) ∈ / {( a , b , 1 )}
as long as ω ≠ 1 \omega\neq 1 ω = 1 .
(c) There is no null vector, i.e.
( 0 , 0 , 0 ) ∉ { ( a , b , 1 ) } . (0,0,0)\notin \{(a,b,1)\}. ( 0 , 0 , 0 ) ∈ / {( a , b , 1 )} .
(d) The inverse does not exist, i.e.
( − a , − b , − 1 ) ∉ { ( a , b , 1 ) } (-a,-b,-1)\notin \{(a,b,1)\} ( − a , − b , − 1 ) ∈ / {( a , b , 1 )}
■ ~\tag*{$\blacksquare$} ■
Exercise 1.1.3 Do functions that vanish at the end points x = 0 x=0 x = 0 and x = L x=L x = L form a vector space? How about periodic functions obeying f ( 0 ) = f ( L ) f(0)=f(L) f ( 0 ) = f ( L ) ? How about functions that obey f ( 0 ) = 4 f(0)=4 f ( 0 ) = 4 ? If the functions do not qualify, list the things that go wrong.
Solution.
(1) { f ( x ) } \{f(x)\} { f ( x )} , f ( 0 ) = f ( L ) = 0 f(0)=f(L)=0 f ( 0 ) = f ( L ) = 0 , form a vector space.
(2) { f ( x ) } \{f(x)\} { f ( x )} , periodic functions obeying f ( 0 ) = f ( L ) f(0)=f(L) f ( 0 ) = f ( L ) , form a vector space. If you want to prove the property of closure in this problem, please mention that f ( x ) + g ( x ) f(x)+g(x) f ( x ) + g ( x ) and α f ( x ) \alpha f(x) α f ( x ) are also periodic functions. That is, f ( 0 ) + g ( 0 ) = f ( L ) + g ( L ) f(0)+g(0)=f(L)+g(L) f ( 0 ) + g ( 0 ) = f ( L ) + g ( L ) , α f ( 0 ) = α f ( L ) \alpha f(0)=\alpha f(L) α f ( 0 ) = α f ( L ) .
(3) { f ( x ) } \{f(x)\} { f ( x )} , f ( 0 ) = 4 f(0)=4 f ( 0 ) = 4 , do not form a vector space, since
(a) If g ( x ) , h ( x ) ∈ { f ( x ) } g(x), h(x)\in \{f(x)\} g ( x ) , h ( x ) ∈ { f ( x )} , then g ( x ) + h ( x ) ∉ { f ( x ) } g(x)+h(x)\notin \{f(x)\} g ( x ) + h ( x ) ∈ / { f ( x )} , since g ( 0 ) + h ( 0 ) = 8 ≠ 4 g(0)+h(0)=8\neq 4 g ( 0 ) + h ( 0 ) = 8 = 4 .
(b) If g ( x ) ∈ { f ( x ) } g(x)\in\{f(x)\} g ( x ) ∈ { f ( x )} , then λ g ( x ) ∉ { f ( x ) } \lambda g(x)\notin \{f(x)\} λ g ( x ) ∈ / { f ( x )} , since λ g ( 0 ) = 4 λ ≠ 4 \lambda g(0)=4\lambda\neq 4 λ g ( 0 ) = 4 λ = 4 , as long as λ ≠ 1 \lambda\neq 1 λ = 1 .
(c) No null vector. g ( x ) ≡ 0 ∉ { f ( x ) } g(x)\equiv 0\notin \{f(x)\} g ( x ) ≡ 0 ∈ / { f ( x )} , since g ( 0 ) = 0 ≠ 4 g(0)=0\neq 4 g ( 0 ) = 0 = 4 .
(d) If g ( x ) ∈ { f ( x ) } g(x)\in \{f(x)\} g ( x ) ∈ { f ( x )} , then the inverse − g ( x ) ∉ { f ( x ) } -g(x)\notin \{f(x)\} − g ( x ) ∈ / { f ( x )} , since − g ( 0 ) = − 4 ≠ 4 -g(0)=-4\neq 4 − g ( 0 ) = − 4 = 4 .
■ ~\tag*{$\blacksquare$} ■
Exercise 1.1.4
Consider three elements from the vector space of real 2 × 2 2 \times 2 2 × 2 matrices:
∣ 1 ⟩ = ( 0 1 0 0 ) ∣ 2 ⟩ = ( 1 1 0 1 ) ∣ 3 ⟩ = ( − 2 − 1 0 − 2 ) |1\rangle=\begin{pmatrix}
0 & 1 \\
0 & 0
\end{pmatrix} \quad|2\rangle=\begin{pmatrix}
1 & 1 \\
0 & 1
\end{pmatrix} \quad|3\rangle=\begin{pmatrix}
-2 & -1 \\
0 & -2
\end{pmatrix} ∣1 ⟩ = ( 0 0 1 0 ) ∣2 ⟩ = ( 1 0 1 1 ) ∣3 ⟩ = ( − 2 0 − 1 − 2 )
Are they linearly independent? Support your answer with details. (Notice we are calling these matrices vectors and using kets to represent them to emphasize their role as elements of a vector space.)
Solution. Suppose α 1 ∣ 1 ⟩ + α 2 ∣ 2 ⟩ + α 3 ∣ 3 ⟩ = 0 \alpha_{1}|1\rangle+\alpha_{2}|2\rangle+\alpha_{3}|3\rangle=0 α 1 ∣1 ⟩ + α 2 ∣2 ⟩ + α 3 ∣3 ⟩ = 0 . We have
( 0 ⋅ α 1 + 1 ⋅ α 2 + ( − 2 ) ⋅ α 3 1 ⋅ α 1 + 1 ⋅ α 2 + ( − 1 ) ⋅ α 3 0 ⋅ α 1 + 0 ⋅ α 2 + 0 ⋅ α 3 0 ⋅ α 1 + 1 ⋅ α 2 + ( − 2 ) ⋅ α 3 ) = ( 0 0 0 0 ) \begin{pmatrix}
0\cdot \alpha_{1}+1\cdot \alpha_{2}+(-2)\cdot \alpha_{3}&1\cdot \alpha_{1}+1\cdot \alpha_{2}+(-1)\cdot \alpha_{3}\\
0\cdot \alpha_{1}+0\cdot \alpha_{2}+0\cdot \alpha_{3}&0\cdot \alpha_{1}+1\cdot \alpha_{2}+(-2)\cdot \alpha_{3}
\end{pmatrix}=
\begin{pmatrix}
0&0\\
0&0
\end{pmatrix} ( 0 ⋅ α 1 + 1 ⋅ α 2 + ( − 2 ) ⋅ α 3 0 ⋅ α 1 + 0 ⋅ α 2 + 0 ⋅ α 3 1 ⋅ α 1 + 1 ⋅ α 2 + ( − 1 ) ⋅ α 3 0 ⋅ α 1 + 1 ⋅ α 2 + ( − 2 ) ⋅ α 3 ) = ( 0 0 0 0 )
⇒ { α 2 − 2 α 3 = 0 α 1 + α 2 − α 3 = 0 \Rightarrow\quad\left\{
\begin{aligned}
\alpha_{2}-2\alpha_{3}=0\\
\alpha_{1}+\alpha_{2}-\alpha_{3}=0
\end{aligned}\right. ⇒ { α 2 − 2 α 3 = 0 α 1 + α 2 − α 3 = 0
⇒ { α 1 = − α 3 α 2 = 2 α 3 \Rightarrow\quad \left\{
\begin{aligned}
\alpha_{1}=-\alpha_{3}\\
\alpha_{2}=2\alpha_{3}
\end{aligned}\right. ⇒ { α 1 = − α 3 α 2 = 2 α 3
It is not necessary for α 1 , α 2 \alpha_{1}, \alpha_{2} α 1 , α 2 and α 3 \alpha_{3} α 3 to be 0 0 0 together. Therefore, ∣ 1 ⟩ |1\rangle ∣1 ⟩ , ∣ 2 ⟩ |2\rangle ∣2 ⟩ and ∣ 3 ⟩ |3\rangle ∣3 ⟩ are linearly dependent.
■ ~\tag*{$\blacksquare$} ■
Exercise 1.1.5 Show that the following row vectors are linearly dependent: ( 1 , 1 , 0 ) (1,1,0) ( 1 , 1 , 0 ) , ( 1 , 0 , 1 ) (1,0,1) ( 1 , 0 , 1 ) , and ( 3 , 2 , 1 ) (3,2,1) ( 3 , 2 , 1 ) . Show the opposite for ( 1 , 1 , 0 ) , ( 1 , 0 , 1 ) (1,1,0),(1,0,1) ( 1 , 1 , 0 ) , ( 1 , 0 , 1 ) , and ( 0 , 1 , 1 ) (0,1,1) ( 0 , 1 , 1 ) .
Solution. Suppose α 1 ( 1 , 1 , 0 ) + α 2 ( 1 , 0 , 1 ) + α 3 ( 3 , 2 , 1 ) = 0 \alpha_{1}(1,1,0)+\alpha_{2}(1,0,1)+\alpha_{3}(3,2,1)=0 α 1 ( 1 , 1 , 0 ) + α 2 ( 1 , 0 , 1 ) + α 3 ( 3 , 2 , 1 ) = 0 . Then
{ α 1 + α 2 + 3 α 3 = 0 α 1 + 2 α 3 = 0 α 2 + α 3 = 0 ⇒ { α 1 = − 2 α 3 α 2 = − α 3 \left\{
\begin{aligned}
\alpha_{1}+\alpha_{2}+3\alpha_{3}=0\\
\alpha_{1}+2\alpha_{3}=0\\
\alpha_{2}+\alpha_{3}=0
\end{aligned}\right.\quad\Rightarrow\quad
\left\{
\begin{aligned}
\alpha_{1}=-2\alpha_{3}\\
\alpha_{2}=-\alpha_{3}
\end{aligned}\right. ⎩ ⎨ ⎧ α 1 + α 2 + 3 α 3 = 0 α 1 + 2 α 3 = 0 α 2 + α 3 = 0 ⇒ { α 1 = − 2 α 3 α 2 = − α 3
When α 3 ≠ 0 \alpha_{3}\neq 0 α 3 = 0 . α 1 , α 2 , α 3 \alpha_{1}, \alpha_{2}, \alpha_{3} α 1 , α 2 , α 3 can have non-zero values. Therefore, ( 1 , 1 , 0 ) (1,1,0) ( 1 , 1 , 0 ) , ( 1 , 0 , 1 ) (1,0,1) ( 1 , 0 , 1 ) , ( 3 , 2 , 1 ) (3,2,1) ( 3 , 2 , 1 ) are linearly dependent.
Suppose α 1 ( 1 , 1 , 0 ) + α 2 ( 1 , 0 , 1 ) + α 3 ( 0 , 1 , 1 ) = 0 \alpha_{1}(1,1,0)+\alpha_{2}(1,0,1)+\alpha_{3}(0,1,1)=0 α 1 ( 1 , 1 , 0 ) + α 2 ( 1 , 0 , 1 ) + α 3 ( 0 , 1 , 1 ) = 0 . Then
{ α 1 + α 2 = 0 α 1 + α 3 = 0 α 2 + α 3 = 0 ⇒ { α 1 = 0 α 2 = 0 α 3 = 0 is the only solution. \left\{
\begin{aligned}
\alpha_{1}+\alpha_{2}=0\\
\alpha_{1}+\alpha_{3}=0\\
\alpha_{2}+\alpha_{3}=0
\end{aligned}\right.\quad\Rightarrow\quad
\left\{
\begin{aligned}
\alpha_{1}=0\\
\alpha_{2}=0\\
\alpha_{3}=0
\end{aligned}\right.~\text{is the only solution.} ⎩ ⎨ ⎧ α 1 + α 2 = 0 α 1 + α 3 = 0 α 2 + α 3 = 0 ⇒ ⎩ ⎨ ⎧ α 1 = 0 α 2 = 0 α 3 = 0 is the only solution.
Therefore, ( 1 , 1 , 0 ) (1,1,0) ( 1 , 1 , 0 ) , ( 1 , 0 , 1 ) (1,0,1) ( 1 , 0 , 1 ) , ( 0 , 1 , 1 ) (0,1,1) ( 0 , 1 , 1 ) are linearly independent.
■ ~\tag*{$\blacksquare$} ■
1.2 Inner Product Spaces
1.3 Dual Spaces and the Dirac Notation
Exercise 1.3.1 Form an orthonormal basis in two dimensions starting with A ⃗ = 3 i ⃗ + 4 j ⃗ \vec{A}=3 \vec{i}+4 \vec{j} A = 3 i + 4 j and B ⃗ = 2 i ⃗ − 6 j ⃗ \vec{B}=2 \vec{i}-6 \vec{j} B = 2 i − 6 j . Can you generate another orthonormal basis starting with these two vectors? If so, produce another.
Solution. Using Gram-Schmidt process here, starting from A ⃗ \vec{A} A .
e ⃗ 1 = A ⃗ ∣ A ⃗ ∣ = 3 i ⃗ + 4 j ⃗ 3 2 + 4 2 = 3 5 i ⃗ + 4 5 j ⃗ \vec{e}_{1}=\frac{\vec{A}}{|\vec{A}|}=\frac{3\vec{i}+4\vec{j}}{\sqrt{3^{2}+4^{2}}}=\frac{3}{5}\vec{i}+\frac{4}{5}\vec{j} e 1 = ∣ A ∣ A = 3 2 + 4 2 3 i + 4 j = 5 3 i + 5 4 j
e ⃗ 2 ′ = B ⃗ − ( B ⃗ ⋅ e ⃗ 1 ) e ⃗ 1 = ( 2 i ⃗ − 6 j ⃗ ) − ( 6 5 − 24 5 ) ( 3 5 i ⃗ + 4 5 j ⃗ ) = ( 2 + 54 25 ) i ⃗ + ( − 6 + 72 25 ) j ⃗ = 104 25 i ⃗ − 78 25 j ⃗ \begin{aligned}
\vec{e}_{2}^{\prime}&=\vec{B}-(\vec{B}\cdot \vec{e}_{1})\vec{e}_{1}\\
&=(2\vec{i}-6\vec{j})-\left(\frac{6}{5}-\frac{24}{5}\right)\left(\frac{3}{5}\vec{i}+\frac{4}{5}\vec{j}\right)\\
&=\left(2+\frac{54}{25}\right)\vec{i}+\left(-6+\frac{72}{25}\right)\vec{j}\\
&=\frac{104}{25}\vec{i}-\frac{78}{25}\vec{j}
\end{aligned} e 2 ′ = B − ( B ⋅ e 1 ) e 1 = ( 2 i − 6 j ) − ( 5 6 − 5 24 ) ( 5 3 i + 5 4 j ) = ( 2 + 25 54 ) i + ( − 6 + 25 72 ) j = 25 104 i − 25 78 j
e ⃗ 2 = e ⃗ 2 ′ ∣ e ⃗ 2 ′ ∣ = 104 25 i ⃗ − 78 25 j ⃗ ( 104 25 ) 2 + ( 78 25 ) 2 = 4 5 i ⃗ − 3 5 j ⃗ \vec{e}_{2}=\frac{\vec{e}_{2}^{\prime}}{|\vec{e}_{2}^{\prime}|}=\frac{\frac{104}{25}\vec{i}-\frac{78}{25}\vec{j}}{\sqrt{\left(\frac{104}{25}\right)^{2}+\left(\frac{78}{25}\right)^{2}}}=\frac{4}{5}\vec{i}-\frac{3}{5}\vec{j} e 2 = ∣ e 2 ′ ∣ e 2 ′ = ( 25 104 ) 2 + ( 25 78 ) 2 25 104 i − 25 78 j = 5 4 i − 5 3 j
Therefore, the new basis is e ⃗ 1 = 3 5 i ⃗ + 4 5 j ⃗ \vec{e}_{1}=\frac{3}{5}\vec{i}+\frac{4}{5}\vec{j} e 1 = 5 3 i + 5 4 j , e ⃗ 2 = 4 5 i ⃗ − 3 5 j ⃗ \vec{e}_{2}=\frac{4}{5}\vec{i}-\frac{3}{5}\vec{j} e 2 = 5 4 i − 5 3 j .
■ ~\tag*{$\blacksquare$} ■
Exercise 1.3.2 Show how to go from the basis
∣ I ⟩ = ( 3 0 0 ) ∣ I I ⟩ = ( 0 1 2 ) ∣ I I I ⟩ = ( 0 2 5 ) |I\rangle=\begin{pmatrix}
3 \\
0 \\
0
\end{pmatrix}
\quad|I I\rangle=
\begin{pmatrix}
0 \\
1 \\
2
\end{pmatrix}\quad|I II\rangle=\begin{pmatrix}
0 \\
2 \\
5
\end{pmatrix} ∣ I ⟩ = ⎝ ⎛ 3 0 0 ⎠ ⎞ ∣ II ⟩ = ⎝ ⎛ 0 1 2 ⎠ ⎞ ∣ III ⟩ = ⎝ ⎛ 0 2 5 ⎠ ⎞
to the orthonormal basis
∣ 1 ⟩ = ( 1 0 0 ) ∣ 2 ⟩ = ( 0 1 / 5 2 / 5 ) ∣ 3 ⟩ = ( 0 − 2 / 5 1 / 5 ) |1\rangle=\begin{pmatrix}
1 \\
0 \\
0
\end{pmatrix}
\quad|2\rangle=\begin{pmatrix}
0 \\
1 / \sqrt{5} \\
2 / \sqrt{5}
\end{pmatrix}\quad|3\rangle=\begin{pmatrix}
0 \\
-2 / \sqrt{5} \\
1 / \sqrt{5}
\end{pmatrix} ∣1 ⟩ = ⎝ ⎛ 1 0 0 ⎠ ⎞ ∣2 ⟩ = ⎝ ⎛ 0 1/ 5 2/ 5 ⎠ ⎞ ∣3 ⟩ = ⎝ ⎛ 0 − 2/ 5 1/ 5 ⎠ ⎞
Solution.
∣ 1 ⟩ = ∣ I ⟩ ⟨ I ∣ I ⟩ = 1 3 ( 3 0 0 ) = ( 1 0 0 ) ∣ 2 ′ ⟩ = ∣ I I ⟩ − ( ⟨ 1 ∣ I I ⟩ ) ∣ 1 ⟩ = ( 0 1 2 ) − 0 ⋅ ( 1 0 0 ) = ( 0 1 2 ) ∣ 2 ⟩ = ∣ 2 ′ ⟩ ⟨ 2 ′ ∣ 2 ′ ⟩ = 1 5 ( 0 1 2 ) = ( 0 1 / 5 2 / 5 ) ∣ 3 ′ ⟩ = ∣ I I I ⟩ − ( ⟨ 1 ∣ I I ⟩ ) ∣ 1 ⟩ − ( ⟨ 2 ∣ I I I ⟩ ) ∣ 2 ⟩ = ( 0 2 5 ) − 0 ⋅ ( 1 0 0 ) − ( 0 + 2 5 + 10 5 ) ( 0 1 / 5 2 / 5 ) = ( 0 2 5 ) − ( 0 12 / 5 24 / 5 ) = ( 0 − 2 / 5 1 / 5 ) ∣ 3 ⟩ = ∣ 3 ′ ⟩ ⟨ 3 ′ ∣ 3 ′ ⟩ = ( 0 − 2 / 5 1 / 5 ) \begin{aligned}
|1\rangle&=\frac{|I\rangle}{\sqrt{\langle I \mid I\rangle}}=\frac{1}{3}\begin{pmatrix}
3 \\
0 \\
0
\end{pmatrix}=\begin{pmatrix}
1 \\
0 \\
0
\end{pmatrix}\\
\left|2^{\prime}\right\rangle & =|II\rangle-(\langle 1 \mid II\rangle)|1\rangle \\
&=\begin{pmatrix}
0 \\
1 \\
2
\end{pmatrix}-0 \cdot\begin{pmatrix}
1 \\
0 \\
0
\end{pmatrix} \\
& =\begin{pmatrix}
0 \\
1 \\
2
\end{pmatrix}\\
|2\rangle&=\frac{\left|2^{\prime}\right\rangle}{\sqrt{\left\langle 2^{\prime} \mid 2^{\prime}\right\rangle}}=\frac{1}{\sqrt{5}}\begin{pmatrix}
0 \\
1 \\
2
\end{pmatrix}=\begin{pmatrix}
0 \\
1/\sqrt{5} \\
2/\sqrt{5}
\end{pmatrix}\\
\left|3^{\prime}\right\rangle & =\mid III \rangle-(\langle 1\mid II \rangle)|1\rangle-(\langle 2\mid III\rangle)|2\rangle \\
& =\begin{pmatrix}
0 \\
2 \\
5
\end{pmatrix}-0 \cdot\begin{pmatrix}
1 \\
0 \\
0
\end{pmatrix}-\left(0+\frac{2}{\sqrt{5}}+\frac{10}{\sqrt{5}}\right)\begin{pmatrix}
0 \\
1/\sqrt{5} \\
2/\sqrt{5}
\end{pmatrix}
\\
& =\begin{pmatrix}
0 \\
2 \\
5
\end{pmatrix}
-\begin{pmatrix}
0 \\
12/5 \\
24/5
\end{pmatrix}
\\
& =\begin{pmatrix}
0 \\
-2/5 \\
1/5
\end{pmatrix}\\
|3\rangle&=\frac{\left|3^{\prime}\right\rangle}{\sqrt{\left\langle 3^{\prime} \mid 3^{\prime}\right\rangle}}=\begin{pmatrix}
0 \\
-2 / \sqrt{5} \\
1 / \sqrt{5}
\end{pmatrix}
\end{aligned} ∣1 ⟩ ∣ 2 ′ ⟩ ∣2 ⟩ ∣ 3 ′ ⟩ ∣3 ⟩ = ⟨ I ∣ I ⟩ ∣ I ⟩ = 3 1 ⎝ ⎛ 3 0 0 ⎠ ⎞ = ⎝ ⎛ 1 0 0 ⎠ ⎞ = ∣ II ⟩ − (⟨ 1 ∣ II ⟩) ∣1 ⟩ = ⎝ ⎛ 0 1 2 ⎠ ⎞ − 0 ⋅ ⎝ ⎛ 1 0 0 ⎠ ⎞ = ⎝ ⎛ 0 1 2 ⎠ ⎞ = ⟨ 2 ′ ∣ 2 ′ ⟩ ∣ 2 ′ ⟩ = 5 1 ⎝ ⎛ 0 1 2 ⎠ ⎞ = ⎝ ⎛ 0 1/ 5 2/ 5 ⎠ ⎞ =∣ III ⟩ − (⟨ 1 ∣ II ⟩) ∣1 ⟩ − (⟨ 2 ∣ III ⟩) ∣2 ⟩ = ⎝ ⎛ 0 2 5 ⎠ ⎞ − 0 ⋅ ⎝ ⎛ 1 0 0 ⎠ ⎞ − ( 0 + 5 2 + 5 10 ) ⎝ ⎛ 0 1/ 5 2/ 5 ⎠ ⎞ = ⎝ ⎛ 0 2 5 ⎠ ⎞ − ⎝ ⎛ 0 12/5 24/5 ⎠ ⎞ = ⎝ ⎛ 0 − 2/5 1/5 ⎠ ⎞ = ⟨ 3 ′ ∣ 3 ′ ⟩ ∣ 3 ′ ⟩ = ⎝ ⎛ 0 − 2/ 5 1/ 5 ⎠ ⎞
■ ~\tag*{$\blacksquare$} ■
Exercise 1.3.3 When will this equality ⟨ V ∣ V ⟩ = ⟨ W ∣ V ⟩ ⟨ V ∣ W ⟩ ∣ W ∣ 2 \langle V \mid V\rangle=\frac{\langle W \mid V\rangle\langle V \mid W\rangle}{|W|^2} ⟨ V ∣ V ⟩ = ∣ W ∣ 2 ⟨ W ∣ V ⟩ ⟨ V ∣ W ⟩ be satisfied? Does this agree with your experience with arrows?
Solution. When ∣ V ⟩ = C ∣ W ⟩ |V\rangle=C|W\rangle ∣ V ⟩ = C ∣ W ⟩ , we have
⟨ V ∣ V ⟩ = ∣ C ∣ 2 ⟨ W ∣ W ⟩ = ∣ C ∣ 2 ∣ W ∣ 2 \langle V\mid V\rangle=|C|^{2}\langle W\mid W\rangle=|C|^{2}|W|^{2} ⟨ V ∣ V ⟩ = ∣ C ∣ 2 ⟨ W ∣ W ⟩ = ∣ C ∣ 2 ∣ W ∣ 2
Also
⟨ W ∣ V ⟩ ⟨ V ∣ W ⟩ = ( C ⟨ W ∣ W ⟩ ) ( C ⋆ ⟨ W ∣ W ⟩ ) = ∣ C ∣ 2 ⟨ W ∣ W ⟩ ⟨ W ∣ W ⟩ = ∣ C ∣ 2 ∣ W ∣ 4 \begin{aligned}
\langle W\mid V\rangle\langle V\mid W\rangle&=(C\langle W\mid W\rangle)(C^{\star}\langle W\mid W\rangle)\\
&=|C|^{2}\langle W\mid W\rangle\langle W\mid W\rangle\\
&=|C|^{2}|W|^{4}
\end{aligned} ⟨ W ∣ V ⟩ ⟨ V ∣ W ⟩ = ( C ⟨ W ∣ W ⟩) ( C ⋆ ⟨ W ∣ W ⟩) = ∣ C ∣ 2 ⟨ W ∣ W ⟩ ⟨ W ∣ W ⟩ = ∣ C ∣ 2 ∣ W ∣ 4
Hence,
⟨ V ∣ V ⟩ = ⟨ W ∣ V ⟩ ⟨ V ∣ W ⟩ ∣ W ∣ 2 \langle V\mid V\rangle=\frac{\langle W\mid V\rangle\langle V\mid W\rangle}{|W|^{2}} ⟨ V ∣ V ⟩ = ∣ W ∣ 2 ⟨ W ∣ V ⟩ ⟨ V ∣ W ⟩
When two arrows are parallel or anti-parallel with each other, the square of their inner product equals to the product of their norms.
■ ~\tag*{$\blacksquare$} ■
Exercise 1.3.4 Prove the triangle inequality starting with ∣ V + W ∣ 2 |V+W|^{2} ∣ V + W ∣ 2 . You must use R e ⟨ V ∣ W ⟩ ⩽ ∣ ⟨ V ∣ W ⟩ ∣ \mathrm{Re}\langle V|W\rangle\leqslant |\langle V|W\rangle| Re ⟨ V ∣ W ⟩ ⩽ ∣ ⟨ V ∣ W ⟩ ∣ and the Schwarz inequality. Show that the final inequality becomes an equality only if ∣ V ⟩ = a ∣ W ⟩ |V\rangle=a|W\rangle ∣ V ⟩ = a ∣ W ⟩ where a a a is a real positive scalar.
Solution. R e ⟨ V ∣ W ⟩ ⩽ ∣ ⟨ V ∣ W ⟩ ∣ ⩽ ∣ V ∣ ∣ W ∣ \mathrm{Re}\langle V\mid W\rangle\leqslant |\langle V\mid W\rangle|\leqslant |V||W| Re ⟨ V ∣ W ⟩ ⩽ ∣ ⟨ V ∣ W ⟩ ∣ ⩽ ∣ V ∣∣ W ∣
⇒ R e ⟨ V ∣ W ⟩ ⩽ 2 ∣ V ∣ ∣ W ∣ \Rightarrow\quad \mathrm{Re}\langle V\mid W\rangle\leqslant 2|V||W| ⇒ Re ⟨ V ∣ W ⟩ ⩽ 2∣ V ∣∣ W ∣
Add ∣ V ∣ 2 + ∣ W ∣ 2 |V|^{2}+|W|^{2} ∣ V ∣ 2 + ∣ W ∣ 2 to both sides of the inequality above, we have
⟨ V ∣ V ⟩ + 2 R e ⟨ V ∣ W ⟩ + ⟨ W ∣ W ⟩ ⩽ ∣ V ∣ 2 + ∣ W ∣ 2 + 2 ∣ V ∣ ∣ W ∣ \langle V\mid V\rangle+2\mathrm{Re}\langle V\mid W\rangle+\langle W\mid W\rangle\leqslant |V|^{2}+|W|^{2}+2|V||W| ⟨ V ∣ V ⟩ + 2 Re ⟨ V ∣ W ⟩ + ⟨ W ∣ W ⟩ ⩽ ∣ V ∣ 2 + ∣ W ∣ 2 + 2∣ V ∣∣ W ∣
LHS = ⟨ V ∣ V ⟩ + ⟨ V ∣ W ⟩ + ⟨ W ∣ V ⟩ + ⟨ W ∣ W ⟩ = ⟨ V + W ∣ V + W ⟩ = ∣ V + W ∣ 2 RHS = ( ∣ V ∣ + ∣ W ∣ ) 2 \begin{aligned}
\text{LHS}&=\langle V\mid V\rangle+\langle V\mid W\rangle+\langle W\mid V\rangle+\langle W\mid W\rangle\\
&=\langle V+W\mid V+W\rangle\\
&=|V+W|^{2}\\
\text{RHS}&=(|V|+|W|)^{2}
\end{aligned} LHS RHS = ⟨ V ∣ V ⟩ + ⟨ V ∣ W ⟩ + ⟨ W ∣ V ⟩ + ⟨ W ∣ W ⟩ = ⟨ V + W ∣ V + W ⟩ = ∣ V + W ∣ 2 = ( ∣ V ∣ + ∣ W ∣ ) 2
Therefore,
∣ V + W ∣ 2 ⩽ ( ∣ V ∣ + ∣ W ∣ ) 2 |V+W|^{2}\leqslant (|V|+|W|)^{2} ∣ V + W ∣ 2 ⩽ ( ∣ V ∣ + ∣ W ∣ ) 2
⇒ ∣ V + W ∣ ⩽ ∣ V ∣ + ∣ W ∣ (the triangular inequality) \Rightarrow\quad |V+W|\leqslant |V|+|W|\tag{the triangular inequality} ⇒ ∣ V + W ∣ ⩽ ∣ V ∣ + ∣ W ∣ ( the triangular inequality )
Attention: We are supposed to prove the equality holds only if ∣ V ⟩ = α ∣ W ⟩ |V\rangle=\alpha|W\rangle ∣ V ⟩ = α ∣ W ⟩ , where α \alpha α is a real number.
The equality holds only if the following two equalities hold:
(a) ⟨ V ∣ W ⟩ = ∣ V ∣ ∣ W ∣ \langle V\mid W\rangle=|V||W| ⟨ V ∣ W ⟩ = ∣ V ∣∣ W ∣
(b) R e ⟨ V ∣ W ⟩ = ∣ ⟨ V ∣ W ⟩ ∣ \mathrm{Re}\langle V\mid W\rangle=|\langle V\mid W\rangle| Re ⟨ V ∣ W ⟩ = ∣ ⟨ V ∣ W ⟩ ∣
From the proof process of the Schwarz inequality in Shankar, we know that equality (a) holds only if
∣ Z ⟩ = ∣ V ⟩ − ⟨ W ∣ V ⟩ ∣ W ∣ 2 ∣ W ⟩ = 0 |Z\rangle=|V\rangle-\frac{\langle W\mid V\rangle}{|W|^{2}}|W\rangle=0 ∣ Z ⟩ = ∣ V ⟩ − ∣ W ∣ 2 ⟨ W ∣ V ⟩ ∣ W ⟩ = 0
which means ∣ V ⟩ |V\rangle ∣ V ⟩ must be able to expressed as α ∣ W ⟩ \alpha |W\rangle α ∣ W ⟩ , where α \alpha α is a number. To prove that α \alpha α must be real, we substitude ∣ V ⟩ = α ∣ W ⟩ |V\rangle=\alpha |W\rangle ∣ V ⟩ = α ∣ W ⟩ into equality (b) above.
⟨ V ∣ W ⟩ = α ⋆ ⟨ V ∣ V ⟩ \langle V\mid W\rangle=\alpha^{\star}\langle V\mid V\rangle ⟨ V ∣ W ⟩ = α ⋆ ⟨ V ∣ V ⟩
To satisfy equality (b), ⟨ V ∣ W ⟩ \langle V\mid W\rangle ⟨ V ∣ W ⟩ must be real. Since ⟨ V ∣ V ⟩ \langle V\mid V\rangle ⟨ V ∣ V ⟩ is real, α ⋆ \alpha^{\star} α ⋆ must be real. Therefore, α \alpha α must be a real number.
■ ~\tag*{$\blacksquare$} ■
1.4 Subspaces
Exercise 1.4.1 In a space V n \mathbb{V}^n V n , prove that the set of all vectors { ∣ V ⊥ 1 ⟩ , ∣ V ⊥ 2 ⟩ , … } \left\{\left|V_{\perp}^1\right\rangle,\left|V_{\perp}^2\right\rangle, \ldots\right\} { ∣ ∣ V ⊥ 1 ⟩ , ∣ ∣ V ⊥ 2 ⟩ , … } , orthogonal to any ∣ V ⟩ ≠ ∣ 0 ⟩ |V\rangle \neq |0\rangle ∣ V ⟩ = ∣0 ⟩ , form a subspace V n − 1 \mathbb{V}^{n-1} V n − 1 .
Solution. Given a vector space V n \mathbb{V}^{n} V n , one can start with an arbitrary vector ∣ V ⟩ ≠ 0 |V\rangle\neq 0 ∣ V ⟩ = 0 and construct n − 1 n-1 n − 1 other vectors orthogonal to this ∣ V ⟩ |V\rangle ∣ V ⟩ through Gram-Schmidt process. Since these n − 1 n-1 n − 1 vectors are linear independent, they span a V n − 1 \mathbb{V}^{n-1} V n − 1 subspace. Now we prove that this subspace V n − 1 \mathbb{V}^{n-1} V n − 1 is the set of all vectors orthogonal to ∣ V ⟩ |V\rangle ∣ V ⟩ .
(1) Every vector in V n − 1 \mathbb{V}^{n-1} V n − 1 is orthogonal to ∣ V ⟩ |V\rangle ∣ V ⟩ : Since every vector in V n − 1 \mathbb{V}^{n-1} V n − 1 can be expressed as a linear combination of the n − 1 n-1 n − 1 vectors we constructed above ∣ V ⊥ ⟩ = ∑ i = 1 n − 1 α i ∣ V i ⟩ |V_{\perp}\rangle=\sum\limits_{i=1}^{n-1}\alpha_{i}|V_{i}\rangle ∣ V ⊥ ⟩ = i = 1 ∑ n − 1 α i ∣ V i ⟩ , and ⟨ V ∣ V ⊥ ⟩ = ∑ i = 1 n − 1 α i ⟨ V ∣ V i ⟩ = 0 \langle V\mid V_{\perp}\rangle=\sum\limits_{i=1}^{n-1}\alpha_{i}\langle V\mid V_{i}\rangle=0 ⟨ V ∣ V ⊥ ⟩ = i = 1 ∑ n − 1 α i ⟨ V ∣ V i ⟩ = 0 , because ⟨ V ∣ V i ⟩ = 0 \langle V\mid V_{i}\rangle=0 ⟨ V ∣ V i ⟩ = 0 for each V i V_{i} V i .
(2) Every vector in V n \mathbb{V}^{n} V n but outside V n − 1 \mathbb{V}^{n-1} V n − 1 is not orthogonal to ∣ V ⟩ |V\rangle ∣ V ⟩ : Since this kind of vectors can be expressed as ∣ W ⟩ = α ∣ V ⟩ + ∑ i = 1 n − 1 α i ∣ V i ⟩ |W\rangle=\alpha|V\rangle+\sum\limits_{i=1}^{n-1}\alpha_{i}|V_{i}\rangle ∣ W ⟩ = α ∣ V ⟩ + i = 1 ∑ n − 1 α i ∣ V i ⟩ , where α ≠ 0 \alpha\neq 0 α = 0 . Therefore ⟨ V ∣ W ⟩ = α ⟨ V ∣ V ⟩ = α ≠ 0 \langle V\mid W\rangle=\alpha\langle V\mid V\rangle=\alpha\neq 0 ⟨ V ∣ W ⟩ = α ⟨ V ∣ V ⟩ = α = 0 .
■ ~\tag*{$\blacksquare$} ■
Exercise 1.4.2 Suppose V 1 n 1 \mathbb{V}_1^{n_1} V 1 n 1 and V 2 n 2 \mathbb{V}_2^{n_2} V 2 n 2 are two subspaces such that any element of V 1 \mathbb{V}_1 V 1 is orthogonal to any element of V 2 \mathbb{V}_2 V 2 . Show that the dimensionality of V 1 ⊕ V 2 \mathbb{V}_1 \oplus \mathbb{V}_2 V 1 ⊕ V 2 is n 1 + n 2 n_1+n_2 n 1 + n 2 . (Hint: Theorem 4.)
Solution. Since V 1 n 1 \mathbb{V}_{1}^{n_{1}} V 1 n 1 and V 2 n 2 \mathbb{V}_{2}^{n_{2}} V 2 n 2 are two subspace orthogonal to each other, we can take the n 1 n_{1} n 1 basis vectors of V 1 n 1 \mathbb{V}_{1}^{n_{1}} V 1 n 1 and the n 2 n_{2} n 2 basis vectors of V 2 n 2 \mathbb{V}_{2}^{n_{2}} V 2 n 2 , and put them together. Because these n 1 + n 2 n_{1}+n_{2} n 1 + n 2 vectors are orthogonal to each other, they can span a V n 1 + n 2 \mathbb{V}^{n_{1}+n_{2}} V n 1 + n 2 subspace. We now prove that this V n 1 + n 2 \mathbb{V}^{n_{1}+n_{2}} V n 1 + n 2 is nothing but the V 1 ⊕ V 2 \mathbb{V}_{1}\oplus \mathbb{V}_{2} V 1 ⊕ V 2 .
(1) Every vector in V n 1 + n 2 \mathbb{V}^{n_{1}+n_{2}} V n 1 + n 2 is in V 1 ⊕ V 2 \mathbb{V}_{1}\oplus \mathbb{V}_{2} V 1 ⊕ V 2 :
Since each vector in V n 1 + n 2 \mathbb{V}^{n_{1}+n_{2}} V n 1 + n 2 can be expressed as ∣ U ⟩ = ∑ i = 1 n 1 α i ∣ V i ⟩ + ∑ j = 1 n 2 β j ∣ W j ⟩ |U\rangle=\sum\limits_{i=1}^{n_{1}}\alpha_{i}|V_{i}\rangle+\sum\limits_{j=1}^{n_{2}}\beta_{j}|W_{j}\rangle ∣ U ⟩ = i = 1 ∑ n 1 α i ∣ V i ⟩ + j = 1 ∑ n 2 β j ∣ W j ⟩ , where { ∣ V i ⟩ } \{|V_{i}\rangle\} { ∣ V i ⟩} , { ∣ W j ⟩ } \{|W_{j}\rangle\} { ∣ W j ⟩} are basis vectors of V 1 \mathbb{V}_{1} V 1 and V 2 \mathbb{V}_{2} V 2 respectively. Notice that ∑ i = 1 n 1 α i ∣ V i ⟩ \sum\limits_{i=1}^{n_{1}}\alpha_{i}|V_{i}\rangle i = 1 ∑ n 1 α i ∣ V i ⟩ is a vector in V 1 \mathbb{V}_{1} V 1 , and ∑ j = 1 n 2 b j ∣ W j ⟩ \sum\limits_{j=1}^{n_{2}}b_{j}|W_{j}\rangle j = 1 ∑ n 2 b j ∣ W j ⟩ is a vector in V 2 \mathbb{V}_{2} V 2 . Therefore ∣ U ⟩ |U\rangle ∣ U ⟩ can be expressed as a combination of vectors from V 1 \mathbb{V}_{1} V 1 and V 2 \mathbb{V}_{2} V 2 . According to the definition of V 1 ⊕ V 2 \mathbb{V}_{1}\oplus \mathbb{V}_{2} V 1 ⊕ V 2 (Definition 12 in Shankar), ∣ U ⟩ ∈ V 1 ⊕ V 2 |U\rangle\in \mathbb{V}_{1}\oplus \mathbb{V}_{2} ∣ U ⟩ ∈ V 1 ⊕ V 2 .
(2) Every vector in V 1 ⊕ V 2 \mathbb{V}_{1}\oplus \mathbb{V}_{2} V 1 ⊕ V 2 is in V n 1 + n 2 \mathbb{V}^{n_{1}+n_{2}} V n 1 + n 2 :
Every vector in V 1 ⊕ V 2 \mathbb{V}_{1}\oplus \mathbb{V}_{2} V 1 ⊕ V 2 can be expressed as ∣ Z ⟩ = C 1 ∣ Z 1 ⟩ + C 2 ∣ Z 2 ⟩ |Z\rangle=C_{1}|Z_{1}\rangle+C_{2}|Z_{2}\rangle ∣ Z ⟩ = C 1 ∣ Z 1 ⟩ + C 2 ∣ Z 2 ⟩ , where ∣ Z 1 ⟩ ∈ V 1 n 1 |Z_{1}\rangle\in\mathbb{V}_{1}^{n_{1}} ∣ Z 1 ⟩ ∈ V 1 n 1 , |Z 2 ⟩ ∈ V 2 n 2 Z_{2}\rangle\in \mathbb{V}_{2}^{n_{2}} Z 2 ⟩ ∈ V 2 n 2 . Therefore, ∣ Z 1 ⟩ = ∑ i = 1 n 1 p i ∣ V i ⟩ |Z_{1}\rangle=\sum\limits_{i=1}^{n_{1}}p_{i}|V_{i}\rangle ∣ Z 1 ⟩ = i = 1 ∑ n 1 p i ∣ V i ⟩ , and ∣ Z 2 ⟩ = ∑ j = 1 n 2 q j ∣ W j ⟩ |Z_{2}\rangle=\sum\limits_{j=1}^{n_{2}}q_{j}|W_{j}\rangle ∣ Z 2 ⟩ = j = 1 ∑ n 2 q j ∣ W j ⟩ . Thus, ∣ Z ⟩ = ∑ i = 1 n 1 C 1 p i ∣ V i ⟩ + ∑ j = 1 n 2 C 2 q j ∣ W j ⟩ |Z\rangle=\sum\limits_{i=1}^{n_{1}}C_{1}p_{i}|V_{i}\rangle+\sum\limits_{j=1}^{n_{2}}C_{2}q_{j}|W_{j}\rangle ∣ Z ⟩ = i = 1 ∑ n 1 C 1 p i ∣ V i ⟩ + j = 1 ∑ n 2 C 2 q j ∣ W j ⟩ , which lies in V n 1 + n 2 \mathbb{V}^{n_{1}+n_{2}} V n 1 + n 2 .
Therefore, by theorem 4 in Shankar, there are n 1 + n 2 n_{1}+n_{2} n 1 + n 2 orthogonal vectors in V 1 ⊕ V 2 \mathbb{V}_{1}\oplus \mathbb{V}_{2} V 1 ⊕ V 2 , so the dimension of V 1 ⊕ V 2 \mathbb{V}_{1}\oplus \mathbb{V}_{2} V 1 ⊕ V 2 is n 1 + n 2 n_{1}+n_{2} n 1 + n 2 .
■ ~\tag*{$\blacksquare$} ■
1.5 Linear Operators
1.6 Matrix Elements of Linear Operators
Exercise 1.6.1 An operator Ω \Omega Ω is given by the matrix
( 0 0 1 1 0 0 0 1 0 ) \begin{pmatrix}
0&0&1\\
1&0&0\\
0&1&0
\end{pmatrix} ⎝ ⎛ 0 1 0 0 0 1 1 0 0 ⎠ ⎞
What is its action?
Solution. To see Ω \Omega Ω 's action, let's act on basis vectors:
Ω ∣ 1 ⟩ = ( 0 0 1 1 0 0 0 1 0 ) ( 1 0 0 ) = ( 0 1 0 ) = ∣ 2 ⟩ Ω ∣ 2 ⟩ = ( 0 0 1 1 0 0 0 1 0 ) ( 0 1 0 ) = ( 0 0 1 ) = ∣ 3 ⟩ Ω ∣ 3 ⟩ = ( 0 0 1 1 0 0 0 1 0 ) ( 0 0 1 ) = ( 1 0 0 ) = ∣ 1 ⟩ \begin{aligned}
\Omega|1\rangle&=\begin{pmatrix}
0&0&1\\
1&0&0\\
0&1&0
\end{pmatrix}\begin{pmatrix}
1\\0\\0
\end{pmatrix}=\begin{pmatrix}
0\\1\\0
\end{pmatrix}=|2\rangle\\
\Omega|2\rangle&=\begin{pmatrix}
0&0&1\\
1&0&0\\
0&1&0
\end{pmatrix}\begin{pmatrix}
0\\1\\0
\end{pmatrix}=\begin{pmatrix}
0\\0\\1
\end{pmatrix}=|3\rangle\\
\Omega|3\rangle&=\begin{pmatrix}
0&0&1\\
1&0&0\\
0&1&0
\end{pmatrix}\begin{pmatrix}
0\\0\\1
\end{pmatrix}=\begin{pmatrix}
1\\0\\0
\end{pmatrix}=|1\rangle
\end{aligned} Ω∣1 ⟩ Ω∣2 ⟩ Ω∣3 ⟩ = ⎝ ⎛ 0 1 0 0 0 1 1 0 0 ⎠ ⎞ ⎝ ⎛ 1 0 0 ⎠ ⎞ = ⎝ ⎛ 0 1 0 ⎠ ⎞ = ∣2 ⟩ = ⎝ ⎛ 0 1 0 0 0 1 1 0 0 ⎠ ⎞ ⎝ ⎛ 0 1 0 ⎠ ⎞ = ⎝ ⎛ 0 0 1 ⎠ ⎞ = ∣3 ⟩ = ⎝ ⎛ 0 1 0 0 0 1 1 0 0 ⎠ ⎞ ⎝ ⎛ 0 0 1 ⎠ ⎞ = ⎝ ⎛ 1 0 0 ⎠ ⎞ = ∣1 ⟩
This is a cyclic permutation of the three basis vectors.
It is equivalent to rotation of the coordinate axis along ( 1 , 1 , 1 ) (1,1,1) ( 1 , 1 , 1 ) by 2 π 3 \frac{2\pi}{3} 3 2 π .
■ ~\tag*{$\blacksquare$} ■
Exercise 1.6.2 Given Ω \Omega Ω and Λ \Lambda Λ are Hermitian what can you say about (1) Ω Λ \Omega \Lambda ΩΛ ; (2) Ω Λ + Λ Ω \Omega \Lambda+\Lambda \Omega ΩΛ + ΛΩ ; (3) [ Ω , Λ ] [\Omega, \Lambda] [ Ω , Λ ] ; and (4) i [ Ω , Λ ] \mathrm{i}[\Omega, \Lambda] i [ Ω , Λ ] ?
Solution.
(1) Not Hermitian: ( Ω Λ ) † = Λ † Ω † = Λ Ω ≠ Ω Λ (\Omega\Lambda)^{\dagger}=\Lambda^{\dagger}\Omega^{\dagger}=\Lambda\Omega\neq \Omega\Lambda ( ΩΛ ) † = Λ † Ω † = ΛΩ = ΩΛ
(2) Hermitian:
( Ω Λ + Λ Ω ) † = ( Ω Λ ) † + ( Λ Ω ) † = Λ † Ω † + Ω † Λ † = Λ Ω + Ω Λ = Ω Λ + Λ Ω \begin{aligned}
(\Omega\Lambda+\Lambda\Omega)^{\dagger}&=(\Omega\Lambda)^{\dagger}+(\Lambda\Omega)^{\dagger}\\
&=\Lambda^{\dagger}\Omega^{\dagger}+\Omega^{\dagger}\Lambda^{\dagger}\\
&=\Lambda\Omega+\Omega\Lambda\\
&=\Omega\Lambda+\Lambda\Omega
\end{aligned} ( ΩΛ + ΛΩ ) † = ( ΩΛ ) † + ( ΛΩ ) † = Λ † Ω † + Ω † Λ † = ΛΩ + ΩΛ = ΩΛ + ΛΩ
(3) Anti-Hermitian:
[ Ω , Λ ] † = ( Ω Λ − Λ Ω ) † = ( Ω Λ ) † − ( Λ Ω ) † = Λ † Ω † − Ω † Λ † = Λ Ω − Ω Λ = − ( Ω Λ − Λ Ω ) = − [ Ω , Λ ] \begin{aligned}
\left[\Omega,\Lambda\right]^{\dagger}&=(\Omega\Lambda-\Lambda\Omega)^{\dagger}\\
&=(\Omega\Lambda)^{\dagger}-(\Lambda\Omega)^{\dagger}\\
&=\Lambda^{\dagger}\Omega^{\dagger}-\Omega^{\dagger}\Lambda^{\dagger}\\
&=\Lambda\Omega-\Omega\Lambda\\
&=-(\Omega\Lambda-\Lambda\Omega)\\
&=-[\Omega,\Lambda]
\end{aligned} [ Ω , Λ ] † = ( ΩΛ − ΛΩ ) † = ( ΩΛ ) † − ( ΛΩ ) † = Λ † Ω † − Ω † Λ † = ΛΩ − ΩΛ = − ( ΩΛ − ΛΩ ) = − [ Ω , Λ ]
(4) Hermitian:
( i [ Ω , Λ ] ) † = − i [ Ω , Λ ] † = − i ⋅ ( − [ Ω , Λ ] ) = [ Ω , Λ ] \begin{aligned}
(\mathrm{i}\left[\Omega,\Lambda\right])^{\dagger}&=-\mathrm{i}\left[\Omega,\Lambda\right]^{\dagger}\\
&=-\mathrm{i}\cdot (-\left[\Omega,\Lambda\right])\\
&=[\Omega,\Lambda]
\end{aligned} ( i [ Ω , Λ ] ) † = − i [ Ω , Λ ] † = − i ⋅ ( − [ Ω , Λ ] ) = [ Ω , Λ ]
■ ~\tag*{$\blacksquare$} ■
Exercise 1.6.3 Show that a product of unitary operator is unitary.
Solution. Suppose U 1 , U 2 U_{1}, U_{2} U 1 , U 2 are unitary, which means that
U 1 † U 1 = I = U 2 † U 2 U_{1}^{\dagger}U_{1}=\mathbb{I}=U_{2}^{\dagger}U_{2} U 1 † U 1 = I = U 2 † U 2
Therefore,
( U 1 U 2 ) † ( U 1 U 2 ) = U 2 † U 1 † U 1 U 2 = U 2 † ( U 1 † U 1 ) U 2 = U 2 † I U 2 = U 2 † U 2 = I \begin{aligned}
(U_{1}U_{2})^{\dagger}(U_{1}U_{2})&=U_{2}^{\dagger}U_{1}^{\dagger}U_{1}U_{2}\\
&=U_{2}^{\dagger}(U_{1}^{\dagger}U_{1})U_{2}\\
&=U_{2}^{\dagger}\mathbb{I}\,U_{2}\\
&=U_{2}^{\dagger}U_{2}\\
&=\mathbb{I}
\end{aligned} ( U 1 U 2 ) † ( U 1 U 2 ) = U 2 † U 1 † U 1 U 2 = U 2 † ( U 1 † U 1 ) U 2 = U 2 † I U 2 = U 2 † U 2 = I
Hence a product of unitary operator is unitary.
■ ~\tag*{$\blacksquare$} ■
Exercise 1.6.4 It is assumed that you know (1) what a determinant is, (2) that det Ω T = det Ω \det\Omega^\mathrm{T}=\det\Omega det Ω T = det Ω (T \mathrm{T} T denotes transpose), (3) that the determinant of a product of matrices is the product of the determinants. [If you do not, verify these properties for a two-dimensional case
Ω = ( α β γ δ ) \Omega=\begin{pmatrix}
\alpha & \beta \\
\gamma & \delta
\end{pmatrix} Ω = ( α γ β δ )
with det Ω = ( α δ − β γ ) \det\Omega=(\alpha \delta-\beta \gamma) det Ω = ( α δ − β γ ) .] Prove that the determinant of a unitary matrix is a complex number of unit modulus.
Solution. Suppose U U U is the unitary matrix, which means that it satisfies
U † U = I U^{\dagger}U=\mathbb{I} U † U = I
Take determinant of the both sides, we get
det ( U † U ) = det ( I ) det ( U † ) det ( U ) = 1 det ( ( U T ) ⋆ ) det ( U ) = 1 ( det ( U T ) ) ⋆ det ( U ) = 1 ( det ( U ) ) ⋆ det ( U ) = 1 ∣ det ( U ) ∣ 2 = 1 ∣ det ( U ) ∣ = 1 \begin{aligned}
\det(U^{\dagger}U)&=\det(\mathbb{I})\\
\det(U^{\dagger})\det(U)&=1\\
\det((U^{\mathrm{T}})^{\star})\det(U)&=1\\
(\det(U^{\mathrm{T}}))^{\star}\det(U)&=1\\
(\det(U))^{\star}\det(U)&=1\\
|\det(U)|^{2}&=1\\
|\det(U)|&=1
\end{aligned} det ( U † U ) det ( U † ) det ( U ) det (( U T ) ⋆ ) det ( U ) ( det ( U T ) ) ⋆ det ( U ) ( det ( U ) ) ⋆ det ( U ) ∣ det ( U ) ∣ 2 ∣ det ( U ) ∣ = det ( I ) = 1 = 1 = 1 = 1 = 1 = 1
Therefore, det ( U ) \det (U) det ( U ) is a complex number of unit modulus.
■ ~\tag*{$\blacksquare$} ■
Exercise 1.6.5 Verify that R ( 1 2 π i ) R\left(\frac{1}{2} \pi \mathbf{i}\right) R ( 2 1 π i ) is unitary (orthogonal) by examining its matrix.
Solution. We know from Example 1.6.1,
R ( 1 2 π i ) = ( 1 0 0 0 0 − 1 0 1 0 ) R\left(\frac{1}{2}\pi \mathbf{i}\right)=\begin{pmatrix}
1&0&0\\
0&0&-1\\
0&1&0
\end{pmatrix} R ( 2 1 π i ) = ⎝ ⎛ 1 0 0 0 0 1 0 − 1 0 ⎠ ⎞
Therefore,
R ( 1 2 π i ) † R ( 1 2 π i ) = ( 1 0 0 0 0 1 0 − 1 0 ) ( 1 0 0 0 0 − 1 0 1 0 ) = ( 1 0 0 0 1 0 0 0 1 ) = I R\left(\frac{1}{2}\pi \mathbf{i}\right)^{\dagger}R\left(\frac{1}{2}\pi \mathbf{i}\right)=\begin{pmatrix}
1&0&0\\
0&0&1\\
0&-1&0
\end{pmatrix}\begin{pmatrix}
1&0&0\\
0&0&-1\\
0&1&0
\end{pmatrix}=\begin{pmatrix}
1&0&0\\
0&1&0\\
0&0&1
\end{pmatrix}=\mathbb{I} R ( 2 1 π i ) † R ( 2 1 π i ) = ⎝ ⎛ 1 0 0 0 0 − 1 0 1 0 ⎠ ⎞ ⎝ ⎛ 1 0 0 0 0 1 0 − 1 0 ⎠ ⎞ = ⎝ ⎛ 1 0 0 0 1 0 0 0 1 ⎠ ⎞ = I
Hence, R ( 1 2 π i ) R\left(\frac{1}{2}\pi \mathbf{i}\right) R ( 2 1 π i ) is unitary.
■ ~\tag*{$\blacksquare$} ■
Exercise 1.6.6 Verify that the following matrices are unitary:
1 2 1 / 2 ( 1 i i 1 ) , 1 2 ( 1 + i 1 − i 1 − i 1 + i ) \frac{1}{2^{1 / 2}}\begin{pmatrix}
1 & \mathrm{i} \\
\mathrm{i} & 1
\end{pmatrix}, \quad \frac{1}{2}\begin{pmatrix}
1+\mathrm{i} & 1-\mathrm{i} \\
1-\mathrm{i} & 1+\mathrm{i}
\end{pmatrix} 2 1/2 1 ( 1 i i 1 ) , 2 1 ( 1 + i 1 − i 1 − i 1 + i )
Verify that the determinant is of the form e i θ e^{\mathrm{i} \theta} e i θ in each case. Are any of the above matrices Hermitian?
Solution.
1 2 1 / 2 ( 1 i i 1 ) \frac{1}{2^{1 / 2}}\begin{pmatrix}
1 & \mathrm{i} \\
\mathrm{i} & 1
\end{pmatrix} 2 1/2 1 ( 1 i i 1 ) is unitary, since
1 2 1 / 2 ( 1 i i 1 ) † ⋅ 1 2 1 / 2 ( 1 i i 1 ) = 1 2 ( 1 − i − i 1 ) ( 1 i i 1 ) = 1 2 ( 2 0 0 2 ) = I \frac{1}{2^{1 / 2}}\begin{pmatrix}
1 & \mathrm{i} \\
\mathrm{i} & 1
\end{pmatrix}^{\dagger}\cdot \frac{1}{2^{1 / 2}}\begin{pmatrix}
1 & \mathrm{i} \\
\mathrm{i} & 1
\end{pmatrix}=\frac{1}{2}\begin{pmatrix}
1 & -\mathrm{i} \\
-\mathrm{i} & 1
\end{pmatrix}\begin{pmatrix}
1 & \mathrm{i} \\
\mathrm{i} & 1
\end{pmatrix}=\frac{1}{2}\begin{pmatrix}
2 & 0 \\
0 & 2
\end{pmatrix}=\mathbb{I} 2 1/2 1 ( 1 i i 1 ) † ⋅ 2 1/2 1 ( 1 i i 1 ) = 2 1 ( 1 − i − i 1 ) ( 1 i i 1 ) = 2 1 ( 2 0 0 2 ) = I
The determinant of 1 2 1 / 2 ( 1 i i 1 ) \frac{1}{2^{1 / 2}}\begin{pmatrix}
1 & \mathrm{i} \\
\mathrm{i} & 1
\end{pmatrix} 2 1/2 1 ( 1 i i 1 ) is of e i θ \mathrm{e}^{\mathrm{i}\theta} e i θ form, since
det [ 1 2 1 / 2 ( 1 i i 1 ) ] = ( 1 2 1 / 2 ) 2 ( 1 ⋅ 1 − i ⋅ i ) = 1 = e i θ , where θ = 2 k π for k ∈ Z \det\left[\frac{1}{2^{1 / 2}}\begin{pmatrix}
1 & \mathrm{i} \\
\mathrm{i} & 1
\end{pmatrix}\right]=\left(\frac{1}{2^{1/2}}\right)^{2}(1\cdot 1-\mathrm{i}\cdot \mathrm{i})=1=\mathrm{e}^{\mathrm{i}\theta},~\text{where}~\theta=2k\pi~\text{for}~k\in\mathbb{Z} det [ 2 1/2 1 ( 1 i i 1 ) ] = ( 2 1/2 1 ) 2 ( 1 ⋅ 1 − i ⋅ i ) = 1 = e i θ , where θ = 2 kπ for k ∈ Z
1 2 1 / 2 ( 1 i i 1 ) \frac{1}{2^{1 / 2}}\begin{pmatrix}
1 & \mathrm{i} \\
\mathrm{i} & 1
\end{pmatrix} 2 1/2 1 ( 1 i i 1 ) is not Hermitian, since
1 2 1 / 2 ( 1 i i 1 ) † = 1 2 1 / 2 ( 1 − i − i 1 ) ≠ 1 2 1 / 2 ( 1 i i 1 ) \frac{1}{2^{1 / 2}}\begin{pmatrix}
1 & \mathrm{i} \\
\mathrm{i} & 1
\end{pmatrix}^{\dagger}=\frac{1}{2^{1 / 2}}\begin{pmatrix}
1 & -\mathrm{i} \\
-\mathrm{i} & 1
\end{pmatrix}\neq \frac{1}{2^{1 / 2}}\begin{pmatrix}
1 & \mathrm{i} \\
\mathrm{i} & 1
\end{pmatrix} 2 1/2 1 ( 1 i i 1 ) † = 2 1/2 1 ( 1 − i − i 1 ) = 2 1/2 1 ( 1 i i 1 )
1 2 ( 1 + i 1 − i 1 − i 1 + i ) \frac{1}{2}\begin{pmatrix}
1+\mathrm{i} & 1-\mathrm{i} \\
1-\mathrm{i} & 1+\mathrm{i}
\end{pmatrix} 2 1 ( 1 + i 1 − i 1 − i 1 + i ) is unitary, since
1 2 ( 1 + i 1 − i 1 − i 1 + i ) † ⋅ 1 2 ( 1 + i 1 − i 1 − i 1 + i ) = 1 4 ( 1 − i 1 + i 1 + i 1 − i ) ( 1 + i 1 − i 1 − i 1 + i ) = 1 4 ( 4 0 0 4 ) = I \begin{aligned}
\frac{1}{2}\begin{pmatrix}
1+\mathrm{i} & 1-\mathrm{i} \\
1-\mathrm{i} & 1+\mathrm{i}
\end{pmatrix}^{\dagger}\cdot\frac{1}{2}\begin{pmatrix}
1+\mathrm{i} & 1-\mathrm{i} \\
1-\mathrm{i} & 1+\mathrm{i}
\end{pmatrix}&=\frac{1}{4}\begin{pmatrix}
1-\mathrm{i} & 1+\mathrm{i} \\
1+\mathrm{i} & 1-\mathrm{i}
\end{pmatrix}\begin{pmatrix}
1+\mathrm{i} & 1-\mathrm{i} \\
1-\mathrm{i} & 1+\mathrm{i}
\end{pmatrix}\\
&=\frac{1}{4}\begin{pmatrix}
4 & 0 \\
0 & 4
\end{pmatrix}\\
&=\mathbb{I}
\end{aligned} 2 1 ( 1 + i 1 − i 1 − i 1 + i ) † ⋅ 2 1 ( 1 + i 1 − i 1 − i 1 + i ) = 4 1 ( 1 − i 1 + i 1 + i 1 − i ) ( 1 + i 1 − i 1 − i 1 + i ) = 4 1 ( 4 0 0 4 ) = I
The determinant of 1 2 ( 1 + i 1 − i 1 − i 1 + i ) \frac{1}{2}\begin{pmatrix}
1+\mathrm{i} & 1-\mathrm{i} \\
1-\mathrm{i} & 1+\mathrm{i}
\end{pmatrix} 2 1 ( 1 + i 1 − i 1 − i 1 + i ) is of e i θ \mathrm{e}^{\mathrm{i}\theta} e i θ form, since
det [ 1 2 ( 1 + i 1 − i 1 − i 1 + i ) ] = ( 1 2 ) 2 [ ( 1 + i ) 2 − ( 1 − i ) 2 ] = i \det\left[\frac{1}{2}\begin{pmatrix}
1+\mathrm{i} & 1-\mathrm{i} \\
1-\mathrm{i} & 1+\mathrm{i}
\end{pmatrix}\right]=\left(\frac{1}{2}\right)^{2}[(1+\mathrm{i})^{2}-(1-\mathrm{i})^{2}]=\mathrm{i} det [ 2 1 ( 1 + i 1 − i 1 − i 1 + i ) ] = ( 2 1 ) 2 [( 1 + i ) 2 − ( 1 − i ) 2 ] = i
where θ = 2 k π + π 2 \theta=2k\pi+\frac{\pi}{2} θ = 2 kπ + 2 π for k ∈ Z k\in\mathbb{Z} k ∈ Z .
1 2 ( 1 + i 1 − i 1 − i 1 + i ) \frac{1}{2}\begin{pmatrix}
1+\mathrm{i} & 1-\mathrm{i} \\
1-\mathrm{i} & 1+\mathrm{i}
\end{pmatrix} 2 1 ( 1 + i 1 − i 1 − i 1 + i ) is not Hermitian, since
1 2 ( 1 + i 1 − i 1 − i 1 + i ) † = 1 2 ( 1 − i 1 + i 1 + i 1 − i ) ≠ 1 2 ( 1 + i 1 − i 1 − i 1 + i ) \frac{1}{2}\begin{pmatrix}
1+\mathrm{i} & 1-\mathrm{i} \\
1-\mathrm{i} & 1+\mathrm{i}
\end{pmatrix}^{\dagger}=\frac{1}{2}\begin{pmatrix}
1-\mathrm{i} & 1+\mathrm{i} \\
1+\mathrm{i} & 1-\mathrm{i}
\end{pmatrix}\neq \frac{1}{2}\begin{pmatrix}
1+\mathrm{i} & 1-\mathrm{i} \\
1-\mathrm{i} & 1+\mathrm{i}
\end{pmatrix} 2 1 ( 1 + i 1 − i 1 − i 1 + i ) † = 2 1 ( 1 − i 1 + i 1 + i 1 − i ) = 2 1 ( 1 + i 1 − i 1 − i 1 + i )
■ ~\tag*{$\blacksquare$} ■
1.7 Active and Passive Transformations
Exercise 1.7.1 The trace of a matrix is defined to be the sum of its diagonal matrix elements
Tr Ω = ∑ i Ω i i \operatorname{Tr} \Omega=\sum_i \Omega_{i i} Tr Ω = i ∑ Ω ii
Show that
(1) Tr ( Ω Λ ) = Tr ( Λ Ω ) \operatorname{Tr}(\Omega \Lambda)=\operatorname{Tr}(\Lambda \Omega) Tr ( ΩΛ ) = Tr ( ΛΩ ) .
(2) Tr ( Ω Λ θ ) = Tr ( Λ θ Ω ) = Tr ( θ Ω Λ ) \operatorname{Tr}(\Omega \Lambda \theta)=\operatorname{Tr}(\Lambda \theta \Omega)=\operatorname{Tr}(\theta \Omega \Lambda) Tr ( ΩΛ θ ) = Tr ( Λ θ Ω ) = Tr ( θ ΩΛ ) (The permutations are \textit{cyclic}).
(3) The trace of an operator is unaffected by a unitary change of basis ∣ i ⟩ → U ∣ i ⟩ |i\rangle \rightarrow U|i\rangle ∣ i ⟩ → U ∣ i ⟩ . [Equivalently, show Tr Ω = Tr ( U † Ω U ) \operatorname{Tr} \Omega=\operatorname{Tr}\left(U^{\dagger} \Omega U\right) Tr Ω = Tr ( U † Ω U ) .]
Solution.
(1) Tr ( Ω Λ ) = ∑ i ( Ω Λ ) i i = ∑ i ∑ j Ω i j Λ j i = ∑ j ∑ i Λ j i Ω i j = ∑ j ( Λ Ω ) j j = Tr ( Λ Ω ) \operatorname{Tr}(\Omega \Lambda)=\sum\limits_{i}(\Omega\Lambda)_{ii}=\sum\limits_{i}\sum\limits_{j}\Omega_{ij}\Lambda_{ji}=\sum\limits_{j}\sum\limits_{i}\Lambda_{ji}\Omega_{ij}=\sum\limits_{j}(\Lambda\Omega)_{jj}=\operatorname{Tr}(\Lambda\Omega) Tr ( ΩΛ ) = i ∑ ( ΩΛ ) ii = i ∑ j ∑ Ω ij Λ ji = j ∑ i ∑ Λ ji Ω ij = j ∑ ( ΛΩ ) jj = Tr ( ΛΩ ) .
(2)
Tr ( Ω Λ θ ) = ∑ i ( Ω Λ θ ) i i = ∑ i ∑ j ∑ k Ω i j Λ j k θ k i = ∑ j ∑ k ∑ i Λ j k θ k i Ω i j = ∑ j ( Λ θ Ω ) j j = Tr ( Λ θ Ω ) = ∑ k ∑ i ∑ j θ k i Ω i j Λ j k = ∑ k ( θ Ω Λ ) k k = Tr ( θ Ω Λ ) \begin{aligned}
\operatorname{Tr}(\Omega\Lambda\theta)&=\sum_{i}(\Omega\Lambda\theta)_{ii}=\sum_{i}\sum_{j}\sum_{k}\Omega_{ij}\Lambda_{jk}\theta_{ki}\\
&=\sum_{j}\sum_{k}\sum_{i}\Lambda_{jk}\theta_{ki}\Omega_{ij}=\sum_{j}(\Lambda\theta\Omega)_{jj}=\operatorname{Tr}(\Lambda\theta\Omega)\\
&=\sum_{k}\sum_{i}\sum_{j}\theta_{ki}\Omega_{ij}\Lambda_{jk}=\sum_{k}(\theta\Omega\Lambda)_{kk}=\operatorname{Tr}(\theta\Omega\Lambda)
\end{aligned} Tr ( ΩΛ θ ) = i ∑ ( ΩΛ θ ) ii = i ∑ j ∑ k ∑ Ω ij Λ jk θ ki = j ∑ k ∑ i ∑ Λ jk θ ki Ω ij = j ∑ ( Λ θ Ω ) jj = Tr ( Λ θ Ω ) = k ∑ i ∑ j ∑ θ ki Ω ij Λ jk = k ∑ ( θ ΩΛ ) kk = Tr ( θ ΩΛ )
(3) Tr ( U † Ω U ) = Tr ( Ω U U † ) = Tr ( Ω I ) = Tr ( Ω ) \operatorname{Tr}(U^{\dagger}\Omega U)=\operatorname{Tr}(\Omega UU^{\dagger})=\operatorname{Tr}(\Omega\mathbb{I})=\operatorname{Tr}(\Omega) Tr ( U † Ω U ) = Tr ( Ω U U † ) = Tr ( Ω I ) = Tr ( Ω ) .
■ ~\tag*{$\blacksquare$} ■
Exercise 1.7.2 Show that the determinant of a matrix is unaffected by a unitary change of basis. [Equivalently show det Ω = det ( U † Ω U ) \det \Omega=\det\left(U^{\dagger} \Omega U\right) det Ω = det ( U † Ω U ) .]
Solution.
det ( U † Ω U ) = det U † det Ω det U = det Ω ( det U † det U ) = det Ω det ( U † U ) = det Ω ⋅ 1 = det Ω . \begin{aligned}
\det\left(U^{\dagger} \Omega U\right) & =\det U^{\dagger} \det \Omega \det U \\
& =\det\Omega\left(\det U^{\dagger} \det U\right) \\
& =\det \Omega \det\left(U^{\dagger} U\right) \\
& =\det \Omega \cdot 1 \\
& =\det \Omega .
\end{aligned} det ( U † Ω U ) = det U † det Ω det U = det Ω ( det U † det U ) = det Ω det ( U † U ) = det Ω ⋅ 1 = det Ω.
■ ~\tag*{$\blacksquare$} ■
1.8 The Eigenvalue Problem
Exercise 1.8.1 (1) Find the eigenvalues and normalized eigenvectors of the matrix
Ω = ( 1 3 1 0 2 0 0 1 4 ) \Omega=\begin{pmatrix}
1&3&1\\
0&2&0\\
0&1&4
\end{pmatrix} Ω = ⎝ ⎛ 1 0 0 3 2 1 1 0 4 ⎠ ⎞
(2) Is the matrix Hermitian? Are the eigenvectors orthogonal?
Solution.
(1) To find the eigenvalues and normalized eigenvectors of the matrix Ω \Omega Ω , we can compute the characteristic equation
det ( Ω − ω I ) = ∣ 1 − ω 3 1 0 2 − ω 0 0 1 4 − ω ∣ = ( 1 − ω ) ( 2 − ω ) ( 4 − ω ) = 0 \det(\Omega-\omega\mathbb{I})=\left|\begin{matrix}
1-\omega & 3 & 1\\
0 & 2-\omega & 0\\
0 & 1 & 4-\omega
\end{matrix}\right|=(1-\omega)(2-\omega)(4-\omega)=0 det ( Ω − ω I ) = ∣ ∣ 1 − ω 0 0 3 2 − ω 1 1 0 4 − ω ∣ ∣ = ( 1 − ω ) ( 2 − ω ) ( 4 − ω ) = 0
So the eigenvalues are
ω = 1 , 2 , 4 \omega=1, 2, 4 ω = 1 , 2 , 4
The eigenvectors corresponding eigenvalues are
ω = 1 : ( 0 3 1 0 1 0 0 1 3 ) ( x 1 x 2 x 3 ) = 0 ⇒ ∣ ω = 1 ⟩ = ( 1 0 0 ) ω = 2 : ( − 1 3 1 0 0 0 0 1 2 ) ( x 1 x 2 x 3 ) = 0 ⇒ ∣ ω = 2 ⟩ = 1 30 ( 5 2 − 1 ) ω = 4 : ( − 3 3 1 0 − 2 0 0 1 0 ) ( x 1 x 2 x 3 ) = 0 ⇒ ∣ ω = 4 ⟩ = 1 10 ( 1 0 3 ) \begin{aligned}
\omega=1: &~\begin{pmatrix}
0&3&1\\
0&1&0\\
0&1&3
\end{pmatrix}\begin{pmatrix}
x_{1}\\
x_{2}\\
x_{3}
\end{pmatrix}=0\quad &\Rightarrow\quad &|\omega=1\rangle=\begin{pmatrix}
1\\0\\0
\end{pmatrix}\\
\omega=2: &~\begin{pmatrix}
-1&3&1\\
0&0&0\\
0&1&2
\end{pmatrix}\begin{pmatrix}
x_{1}\\
x_{2}\\
x_{3}
\end{pmatrix}=0\quad &\Rightarrow\quad &|\omega=2\rangle=\frac{1}{\sqrt{30}}\begin{pmatrix}
5\\2\\-1
\end{pmatrix}\\
\omega=4: &~\begin{pmatrix}
-3&3&1\\
0&-2&0\\
0&1&0
\end{pmatrix}\begin{pmatrix}
x_{1}\\
x_{2}\\
x_{3}
\end{pmatrix}=0\quad &\Rightarrow\quad &|\omega=4\rangle=\frac{1}{\sqrt{10}}\begin{pmatrix}
1\\0\\3
\end{pmatrix}
\end{aligned} ω = 1 : ω = 2 : ω = 4 : ⎝ ⎛ 0 0 0 3 1 1 1 0 3 ⎠ ⎞ ⎝ ⎛ x 1 x 2 x 3 ⎠ ⎞ = 0 ⎝ ⎛ − 1 0 0 3 0 1 1 0 2 ⎠ ⎞ ⎝ ⎛ x 1 x 2 x 3 ⎠ ⎞ = 0 ⎝ ⎛ − 3 0 0 3 − 2 1 1 0 0 ⎠ ⎞ ⎝ ⎛ x 1 x 2 x 3 ⎠ ⎞ = 0 ⇒ ⇒ ⇒ ∣ ω = 1 ⟩ = ⎝ ⎛ 1 0 0 ⎠ ⎞ ∣ ω = 2 ⟩ = 30 1 ⎝ ⎛ 5 2 − 1 ⎠ ⎞ ∣ ω = 4 ⟩ = 10 1 ⎝ ⎛ 1 0 3 ⎠ ⎞
(2) Matrix Ω \Omega Ω is not Hermitian, since
Ω † = ( 1 0 0 3 2 0 1 0 4 ) ≠ Ω \Omega^{\dagger}=\begin{pmatrix}
1&0&0\\
3&2&0\\
1&0&4
\end{pmatrix}\neq\Omega Ω † = ⎝ ⎛ 1 3 1 0 2 0 0 0 4 ⎠ ⎞ = Ω
The eigenvectors are not orthogonal, since
⟨ ω = 1 ∣ ω = 2 ⟩ = 1 × 5 30 + 0 × 2 30 + 0 × − 1 30 = 30 6 ≠ 0 ⟨ ω = 1 ∣ ω = 4 ⟩ = 1 × 1 10 + 0 × 0 + 0 × 3 10 = 10 10 ≠ 0 ⟨ ω = 2 ∣ ω = 4 ⟩ = 5 30 × 1 10 + 2 30 × 0 + − 1 30 × 3 10 = 3 15 ≠ 0 \begin{aligned}
&\langle\omega=1\mid\omega=2\rangle=1\times \frac{5}{\sqrt{30}}+0\times\frac{2}{\sqrt{30}}+0\times\frac{-1}{\sqrt{30}}=\frac{\sqrt{30}}{6}\neq 0\\ &\langle\omega=1\mid\omega=4\rangle=1\times\frac{1}{\sqrt{10}}+0\times 0+0\times\frac{3}{\sqrt{10}}=\frac{\sqrt{10}}{10}\neq 0\\&\langle\omega=2\mid\omega=4\rangle=\frac{5}{\sqrt{30}}\times\frac{1}{\sqrt{10}}+\frac{2}{\sqrt{30}}\times 0+\frac{-1}{\sqrt{30}}\times\frac{3}{\sqrt{10}}=\frac{\sqrt{3}}{15}\neq 0
\end{aligned} ⟨ ω = 1 ∣ ω = 2 ⟩ = 1 × 30 5 + 0 × 30 2 + 0 × 30 − 1 = 6 30 = 0 ⟨ ω = 1 ∣ ω = 4 ⟩ = 1 × 10 1 + 0 × 0 + 0 × 10 3 = 10 10 = 0 ⟨ ω = 2 ∣ ω = 4 ⟩ = 30 5 × 10 1 + 30 2 × 0 + 30 − 1 × 10 3 = 15 3 = 0
■ ~\tag*{$\blacksquare$} ■
Exercise 1.8.2 Consider the matrix
Ω = ( 0 0 1 0 0 0 1 0 0 ) \Omega=\begin{pmatrix}
0&0&1\\
0&0&0\\
1&0&0
\end{pmatrix} Ω = ⎝ ⎛ 0 0 1 0 0 0 1 0 0 ⎠ ⎞
(1) Is it Hermitian?
(2) Find its eigenvalues and eigenvectors.
(3) Verify that U † Ω U U^{\dagger} \Omega U U † Ω U is diagonal, U U U being the matrix of eigenvectors of Ω \Omega Ω .
Solution.
(1) Matrix Ω \Omega Ω is Hermitian since
Ω † = Ω \Omega^{\dagger}=\Omega Ω † = Ω
(2) To find the eigenvalues and eigenvectors of the matrix Ω \Omega Ω , we can compute the characteristic equation
det ( Ω − ω I ) = ∣ − ω 0 1 0 − ω 0 1 0 − ω ∣ = − ω 3 + ω = − ω ( ω + 1 ) ( ω − 1 ) = 0 \det(\Omega-\omega\mathbb{I})=\left|\begin{matrix}
-\omega & 0 & 1\\
0 & -\omega & 0\\
1 & 0 & -\omega
\end{matrix}\right|=-\omega^{3}+\omega=-\omega(\omega+1)(\omega-1)=0 det ( Ω − ω I ) = ∣ ∣ − ω 0 1 0 − ω 0 1 0 − ω ∣ ∣ = − ω 3 + ω = − ω ( ω + 1 ) ( ω − 1 ) = 0
Therefore, eigenvalues are
ω = − 1 , 0 , 1 \omega=-1, 0, 1 ω = − 1 , 0 , 1
The eigenvectors corresponding to eigenvalues are
ω = − 1 : ( 1 0 1 0 1 0 1 0 1 ) ( x 1 x 2 x 3 ) = 0 ⇒ ∣ ω = − 1 ⟩ = 1 2 ( 1 0 − 1 ) ω = 0 : ( 0 0 1 0 0 0 1 0 0 ) ( x 1 x 2 x 3 ) = 0 ⇒ ∣ ω = 0 ⟩ = ( 0 1 0 ) ω = 1 : ( − 1 0 1 0 − 1 0 1 0 − 1 ) ( x 1 x 2 x 3 ) = 0 ⇒ ∣ ω = 1 ⟩ = 1 2 ( 1 0 1 ) \begin{aligned}
\omega=-1: &~\begin{pmatrix}
1&0&1\\
0&1&0\\
1&0&1
\end{pmatrix}\begin{pmatrix}
x_{1}\\
x_{2}\\
x_{3}
\end{pmatrix}=0\quad &\Rightarrow\quad &|\omega=-1\rangle=\frac{1}{\sqrt{2}}\begin{pmatrix}
1\\0\\-1
\end{pmatrix}\\
\omega=0: &~\begin{pmatrix}
0&0&1\\
0&0&0\\
1&0&0
\end{pmatrix}\begin{pmatrix}
x_{1}\\
x_{2}\\
x_{3}
\end{pmatrix}=0\quad &\Rightarrow\quad &|\omega=0\rangle=\begin{pmatrix}
0\\1\\0
\end{pmatrix}\\
\omega=1: &~\begin{pmatrix}
-1&0&1\\
0&-1&0\\
1&0&-1
\end{pmatrix}\begin{pmatrix}
x_{1}\\
x_{2}\\
x_{3}
\end{pmatrix}=0\quad &\Rightarrow\quad &|\omega=1\rangle=\frac{1}{\sqrt{2}}\begin{pmatrix}
1\\0\\1
\end{pmatrix}
\end{aligned} ω = − 1 : ω = 0 : ω = 1 : ⎝ ⎛ 1 0 1 0 1 0 1 0 1 ⎠ ⎞ ⎝ ⎛ x 1 x 2 x 3 ⎠ ⎞ = 0 ⎝ ⎛ 0 0 1 0 0 0 1 0 0 ⎠ ⎞ ⎝ ⎛ x 1 x 2 x 3 ⎠ ⎞ = 0 ⎝ ⎛ − 1 0 1 0 − 1 0 1 0 − 1 ⎠ ⎞ ⎝ ⎛ x 1 x 2 x 3 ⎠ ⎞ = 0 ⇒ ⇒ ⇒ ∣ ω = − 1 ⟩ = 2 1 ⎝ ⎛ 1 0 − 1 ⎠ ⎞ ∣ ω = 0 ⟩ = ⎝ ⎛ 0 1 0 ⎠ ⎞ ∣ ω = 1 ⟩ = 2 1 ⎝ ⎛ 1 0 1 ⎠ ⎞
(3) If U U U is the matrix of eigenvectors of Ω \Omega Ω , then
U = ( 1 2 0 1 2 0 1 0 − 1 2 0 1 2 ) U † = ( 1 2 0 − 1 2 0 1 0 1 2 0 1 2 ) U=\begin{pmatrix}
\frac{1}{\sqrt{2}} & 0 & \frac{1}{\sqrt{2}}\\
0 & 1 & 0\\
-\frac{1}{\sqrt{2}} & 0 & \frac{1}{\sqrt{2}}
\end{pmatrix}\qquad
U^{\dagger}=\begin{pmatrix}
\frac{1}{\sqrt{2}} & 0 & -\frac{1}{\sqrt{2}}\\
0 & 1 & 0\\
\frac{1}{\sqrt{2}} & 0 & \frac{1}{\sqrt{2}}
\end{pmatrix} U = ⎝ ⎛ 2 1 0 − 2 1 0 1 0 2 1 0 2 1 ⎠ ⎞ U † = ⎝ ⎛ 2 1 0 2 1 0 1 0 − 2 1 0 2 1 ⎠ ⎞
We can compute
U † Ω U = ( 1 2 0 − 1 2 0 1 0 1 2 0 1 2 ) ( 0 0 1 0 0 0 1 0 0 ) ( 1 2 0 1 2 0 1 0 − 1 2 0 1 2 ) = ( − 1 2 0 1 2 0 0 0 1 2 0 1 2 ) ( 1 2 0 1 2 0 1 0 − 1 2 0 1 2 ) = ( − 1 0 0 0 0 0 0 0 1 ) \begin{aligned}
U^{\dagger}\Omega U&=\begin{pmatrix}
\frac{1}{\sqrt{2}} & 0 & -\frac{1}{\sqrt{2}}\\
0 & 1 & 0\\
\frac{1}{\sqrt{2}} & 0 & \frac{1}{\sqrt{2}}
\end{pmatrix}\begin{pmatrix}
0&0&1\\
0&0&0\\
1&0&0
\end{pmatrix}\begin{pmatrix}
\frac{1}{\sqrt{2}} & 0 & \frac{1}{\sqrt{2}}\\
0 & 1 & 0\\
-\frac{1}{\sqrt{2}} & 0 & \frac{1}{\sqrt{2}}
\end{pmatrix}\\
&=\begin{pmatrix}
-\frac{1}{\sqrt{2}} & 0 & \frac{1}{\sqrt{2}}\\
0 & 0 & 0\\
\frac{1}{\sqrt{2}} & 0 & \frac{1}{\sqrt{2}}
\end{pmatrix}\begin{pmatrix}
\frac{1}{\sqrt{2}} & 0 & \frac{1}{\sqrt{2}}\\
0 & 1 & 0\\
-\frac{1}{\sqrt{2}} & 0 & \frac{1}{\sqrt{2}}
\end{pmatrix}\\
&=\begin{pmatrix}
-1 & 0 & 0\\
0 & 0 & 0\\
0 & 0 & 1
\end{pmatrix}
\end{aligned} U † Ω U = ⎝ ⎛ 2 1 0 2 1 0 1 0 − 2 1 0 2 1 ⎠ ⎞ ⎝ ⎛ 0 0 1 0 0 0 1 0 0 ⎠ ⎞ ⎝ ⎛ 2 1 0 − 2 1 0 1 0 2 1 0 2 1 ⎠ ⎞ = ⎝ ⎛ − 2 1 0 2 1 0 0 0 2 1 0 2 1 ⎠ ⎞ ⎝ ⎛ 2 1 0 − 2 1 0 1 0 2 1 0 2 1 ⎠ ⎞ = ⎝ ⎛ − 1 0 0 0 0 0 0 0 1 ⎠ ⎞
This is a diagonal matrix.
■ ~\tag*{$\blacksquare$} ■
Exercise 1.8.3 Consider the Hermitian matrix
Ω = 1 2 ( 2 0 0 0 3 − 1 0 − 1 3 ) \Omega=\frac{1}{2}\begin{pmatrix}
2 & 0 & 0\\
0 & 3 & -1\\
0 & -1 & 3
\end{pmatrix} Ω = 2 1 ⎝ ⎛ 2 0 0 0 3 − 1 0 − 1 3 ⎠ ⎞
(1) Show that ω 1 = ω 2 = 1 \omega_{1}=\omega_{2}=1 ω 1 = ω 2 = 1 ; ω 3 = 2 \omega_{3}=2 ω 3 = 2 .
(2) Show that ∣ ω = 2 ⟩ |\omega=2\rangle ∣ ω = 2 ⟩ is any vector of the form
1 ( 2 a 2 ) 1 / 2 ( 0 a − a ) \frac{1}{(2a^{2})^{1/2}}\begin{pmatrix}
0\\a\\-a
\end{pmatrix} ( 2 a 2 ) 1/2 1 ⎝ ⎛ 0 a − a ⎠ ⎞
(3) Show that the ω = 1 \omega=1 ω = 1 eigenspace contains all vectors of the form
1 ( b 2 + 2 c 2 ) 1 / 2 ( b c c ) \frac{1}{(b^{2}+2c^{2})^{1/2}}\begin{pmatrix}
b\\c\\c
\end{pmatrix} ( b 2 + 2 c 2 ) 1/2 1 ⎝ ⎛ b c c ⎠ ⎞
either by feeding ω = 1 \omega=1 ω = 1 into the equations or by requiring that the ω = 1 \omega=1 ω = 1 eigenspace be orthogonal to ∣ ω = 2 ⟩ |\omega=2\rangle ∣ ω = 2 ⟩ .
Solution. (1) The characteristic equation is
det ( Ω − ω I ) = ∣ 1 − ω 0 0 0 3 2 − ω − 1 2 0 − 1 2 3 2 − ω ∣ = ( 1 − ω ) ( 3 2 − ω ) 2 − ( 1 − ω ) ( − 1 2 ) 2 = ( 1 − ω ) [ ( 3 2 − ω ) 2 − 1 4 ] = ( 1 − ω ) ( ω 2 − 3 ω + 2 ) = ( 1 − ω ) ( ω − 1 ) ( ω − 2 ) = 0 \begin{aligned}
\det(\Omega-\omega\mathbb{I})&=\left|\begin{matrix}
1-\omega & 0 & 0\\
0 & \frac{3}{2}-\omega & -\frac{1}{2}\\
0 & -\frac{1}{2} & \frac{3}{2}-\omega
\end{matrix}\right|\\
&=(1-\omega)\left(\frac{3}{2}-\omega\right)^{2}-(1-\omega)\left(-\frac{1}{2}\right)^{2}\\
&=(1-\omega)\left[\left(\frac{3}{2}-\omega\right)^{2}-\frac{1}{4}\right]=(1-\omega)(\omega^{2}-3\omega+2)\\
&=(1-\omega)(\omega-1)(\omega-2)=0
\end{aligned} det ( Ω − ω I ) = ∣ ∣ 1 − ω 0 0 0 2 3 − ω − 2 1 0 − 2 1 2 3 − ω ∣ ∣ = ( 1 − ω ) ( 2 3 − ω ) 2 − ( 1 − ω ) ( − 2 1 ) 2 = ( 1 − ω ) [ ( 2 3 − ω ) 2 − 4 1 ] = ( 1 − ω ) ( ω 2 − 3 ω + 2 ) = ( 1 − ω ) ( ω − 1 ) ( ω − 2 ) = 0
Then the eigenvalues are
ω 1 = ω 2 = 1 ω 3 = 2 \omega_{1}=\omega_{2}=1\quad\omega_{3}=2 ω 1 = ω 2 = 1 ω 3 = 2
(2) To get the eigenvector corresponding to eigenvalue ω = 2 \omega=2 ω = 2 , we need to solve the equation
( − 1 0 0 0 − 1 2 − 1 2 0 − 1 2 − 1 2 ) ( x 1 x 2 x 3 ) = 0 ⇒ { x 1 = 0 x 2 + x 3 = 0 \begin{pmatrix}
-1 & 0 & 0\\
0 & -\frac{1}{2} & -\frac{1}{2}\\
0 & -\frac{1}{2} & -\frac{1}{2}
\end{pmatrix}\begin{pmatrix}
x_{1}\\x_{2}\\x_{3}
\end{pmatrix}=0\quad \Rightarrow \quad
\left\{
\begin{aligned}
x_{1}&=0\\
x_{2}+x_{3}&=0
\end{aligned}
\right. ⎝ ⎛ − 1 0 0 0 − 2 1 − 2 1 0 − 2 1 − 2 1 ⎠ ⎞ ⎝ ⎛ x 1 x 2 x 3 ⎠ ⎞ = 0 ⇒ { x 1 x 2 + x 3 = 0 = 0
Set x 2 = a x_{2}=a x 2 = a , we have x 3 = − a x_{3}=-a x 3 = − a . Therefore,
∣ ω = 2 ⟩ = 1 2 a 2 ( 0 a − a ) |\omega=2\rangle=\frac{1}{\sqrt{2a^{2}}}\begin{pmatrix}
0\\a\\-a
\end{pmatrix} ∣ ω = 2 ⟩ = 2 a 2 1 ⎝ ⎛ 0 a − a ⎠ ⎞
(3) For ω = 1 \omega=1 ω = 1 :
( 0 0 0 0 1 2 − 1 2 0 − 1 2 1 2 ) ( x 1 x 2 x 3 ) = 0 \begin{pmatrix}
0 & 0 & 0\\
0 & \frac{1}{2} & -\frac{1}{2}\\
0 & -\frac{1}{2} & \frac{1}{2}
\end{pmatrix}
\begin{pmatrix}
x_{1}\\x_{2}\\x_{3}
\end{pmatrix}=0 ⎝ ⎛ 0 0 0 0 2 1 − 2 1 0 − 2 1 2 1 ⎠ ⎞ ⎝ ⎛ x 1 x 2 x 3 ⎠ ⎞ = 0
x 1 x_{1} x 1 is arbitrary, set x 1 = b x_{1}=b x 1 = b . x 2 − x 3 = 0 x_{2}-x_{3}=0 x 2 − x 3 = 0 , set x 2 = c x_{2}=c x 2 = c , then x 3 = c x_{3}=c x 3 = c . Therefore, the eigenvector corresponding ω = 1 \omega=1 ω = 1 is of the form
∣ ω = 1 ⟩ = 1 b 2 + 2 c 2 ( b c c ) . |\omega=1\rangle=\frac{1}{\sqrt{b^{2}+2c^{2}}}\begin{pmatrix}
b\\c\\c
\end{pmatrix}. ∣ ω = 1 ⟩ = b 2 + 2 c 2 1 ⎝ ⎛ b c c ⎠ ⎞ .
■ ~\tag*{$\blacksquare$} ■
Exercise 1.8.4 An arbitrary n × n n \times n n × n matrix need not have n n n eigenvectors. Consider as an example
Ω = ( 4 1 − 1 2 ) \Omega=\begin{pmatrix}
4 & 1 \\
-1 & 2
\end{pmatrix} Ω = ( 4 − 1 1 2 )
(1) Show that ω 1 = ω 2 = 3 \omega_1=\omega_2=3 ω 1 = ω 2 = 3 .
(2) By feeding in this value show we get only one eigenvector of the form
1 ( 2 a 2 ) 1 / 2 ( + a − a ) \frac{1}{\left(2 a^2\right)^{1 / 2}}\begin{pmatrix}
+a \\
-a
\end{pmatrix} ( 2 a 2 ) 1/2 1 ( + a − a )
We cannot find another one that is linear independent.
Solution. (1) The characteristic equation is
det ( Ω − ω I ) = ∣ 4 − ω 1 − 1 2 − ω ∣ = ( 4 − ω ) ( 2 − ω ) + 1 = ω 2 − 6 ω + 9 = 0 \det(\Omega-\omega\mathbb{I})=\left|\begin{matrix}
4-\omega & 1\\
-1 & 2-\omega
\end{matrix}\right|=(4-\omega)(2-\omega)+1=\omega^{2}-6\omega+9=0 det ( Ω − ω I ) = ∣ ∣ 4 − ω − 1 1 2 − ω ∣ ∣ = ( 4 − ω ) ( 2 − ω ) + 1 = ω 2 − 6 ω + 9 = 0
Thus the eigenvalues are
ω 1 = ω 2 = 3 \omega_{1}=\omega_{2}=3 ω 1 = ω 2 = 3
(2) By feeding this eigenvalue ω = 3 \omega=3 ω = 3 , we get the equation
( 1 1 − 1 − 1 ) ( x 1 x 2 ) = 0 ⇒ x 1 + x 2 = 0 \begin{pmatrix}
1 & 1\\
-1 & -1
\end{pmatrix}
\begin{pmatrix}
x_{1}\\x_{2}
\end{pmatrix}=0\qquad \Rightarrow \qquad
x_{1}+x_{2}=0 ( 1 − 1 1 − 1 ) ( x 1 x 2 ) = 0 ⇒ x 1 + x 2 = 0
Set x 1 = a x_{1}=a x 1 = a , we have x 2 = − a x_{2}=-a x 2 = − a . Therefore, the eigenvector is of the form
∣ ω = 3 ⟩ = 1 2 a 2 ( a − a ) . |\omega=3\rangle=\frac{1}{\sqrt{2a^{2}}}\begin{pmatrix}
a\\-a
\end{pmatrix}. ∣ ω = 3 ⟩ = 2 a 2 1 ( a − a ) .
This is the only eigenvector we can find.
■ ~\tag*{$\blacksquare$} ■
Exercise 1.8.5 Consider the matrix
Ω = ( cos θ sin θ − sin θ cos θ ) \Omega=\begin{pmatrix}
\cos \theta & \sin \theta \\
-\sin \theta & \cos \theta
\end{pmatrix} Ω = ( cos θ − sin θ sin θ cos θ )
(1) Show that it is unitary.
(2) Show that its eigenvalues are e i θ \mathrm{e}^{\mathrm{i} \theta} e i θ and e − i θ \mathrm{e}^{-\mathrm{i} \theta} e − i θ .
(3) Find the corresponding eigenvectors; show that they are orthogonal.
(4) Verify that U † Ω U = U^{\dagger} \Omega U= U † Ω U = (diagonal matrix), where U U U is the matrix of eigenvectors of Ω \Omega Ω .
Solution. (1) Matrix Ω \Omega Ω is unitary, since
Ω † Ω = ( cos θ − sin θ sin θ cos θ ) ( cos θ sin θ − sin θ cos θ ) = ( cos 2 θ + sin 2 θ cos θ sin θ − sin θ cos θ sin θ cos θ − cos θ sin θ sin 2 θ + cos 2 θ ) = ( 1 0 0 1 ) = I \begin{aligned}
\Omega^{\dagger}\Omega&=\begin{pmatrix}
\cos \theta & -\sin \theta \\
\sin \theta & \cos \theta
\end{pmatrix}\begin{pmatrix}
\cos \theta & \sin \theta \\
-\sin \theta & \cos \theta
\end{pmatrix}\\
&=\begin{pmatrix}
\cos^{2}\theta+\sin^{2}\theta & \cos\theta\sin\theta-\sin\theta\cos\theta\\
\sin\theta\cos\theta-\cos\theta\sin\theta & \sin^{2}\theta+\cos^{2}\theta
\end{pmatrix}\\
&=\begin{pmatrix}
1 & 0\\
0 & 1
\end{pmatrix}\\
&=\mathbb{I}
\end{aligned} Ω † Ω = ( cos θ sin θ − sin θ cos θ ) ( cos θ − sin θ sin θ cos θ ) = ( cos 2 θ + sin 2 θ sin θ cos θ − cos θ sin θ cos θ sin θ − sin θ cos θ sin 2 θ + cos 2 θ ) = ( 1 0 0 1 ) = I
(2) Solve the characteristic equation
det ( Ω − ω I ) = ∣ cos θ − ω sin θ − sin θ cos θ − ω ∣ = ω 2 − 2 ω cos θ + 1 = 0 \det(\Omega-\omega\mathbb{I})=\left|\begin{matrix}
\cos\theta-\omega & \sin\theta\\
-\sin\theta & \cos\theta-\omega
\end{matrix}\right|=\omega^{2}-2\omega\cos\theta+1=0 det ( Ω − ω I ) = ∣ ∣ cos θ − ω − sin θ sin θ cos θ − ω ∣ ∣ = ω 2 − 2 ω cos θ + 1 = 0
By Euler's formula, we get the eigenvalues
ω = cos θ ± i sin θ = e ± i θ \omega=\cos\theta\pm \mathrm{i}\sin\theta=\mathrm{e}^{\pm\mathrm{i}\theta} ω = cos θ ± i sin θ = e ± i θ
(3) By feeding this eigenvalue, we get the equations
ω = e − i θ : ( i sin θ sin θ − sin θ i sin θ ) ( x 1 x 2 ) = 0 ⇒ i x 1 + x 2 = 0 ω = e i θ : ( − i sin θ sin θ − sin θ − i sin θ ) ( x 1 x 2 ) = 0 ⇒ − i x 1 + x 2 = 0 \begin{aligned}
\omega&=\mathrm{e}^{-\mathrm{i}\theta}: &~\begin{pmatrix}
\mathrm{i}\sin\theta & \sin\theta\\
-\sin\theta &\mathrm{i} \sin\theta
\end{pmatrix}\begin{pmatrix}
x_{1}\\x_{2}
\end{pmatrix}=0\quad &\Rightarrow\quad \mathrm{i}x_{1}+x_{2}=0\\
\omega&=\mathrm{e}^{\mathrm{i}\theta}: &~\begin{pmatrix}
-\mathrm{i}\sin\theta & \sin\theta\\
-\sin\theta &-\mathrm{i} \sin\theta
\end{pmatrix}\begin{pmatrix}
x_{1}\\x_{2}
\end{pmatrix}=0\quad &\Rightarrow\quad -\mathrm{i}x_{1}+x_{2}=0
\end{aligned} ω ω = e − i θ : = e i θ : ( i sin θ − sin θ sin θ i sin θ ) ( x 1 x 2 ) = 0 ( − i sin θ − sin θ sin θ − i sin θ ) ( x 1 x 2 ) = 0 ⇒ i x 1 + x 2 = 0 ⇒ − i x 1 + x 2 = 0
Thus the corresponding eigenvectors are
∣ ω = e − i θ ⟩ = 1 2 ( 1 − i ) ∣ ω = e i θ ⟩ = 1 2 ( 1 i ) \begin{aligned}
&|\omega=\mathrm{e}^{-\mathrm{i}\theta}\rangle=\frac{1}{\sqrt{2}}\begin{pmatrix}
1\\-\mathrm{i}
\end{pmatrix}\\
&|\omega=\mathrm{e}^{\mathrm{i}\theta}\rangle=\frac{1}{\sqrt{2}}\begin{pmatrix}
1\\\mathrm{i}
\end{pmatrix}
\end{aligned} ∣ ω = e − i θ ⟩ = 2 1 ( 1 − i ) ∣ ω = e i θ ⟩ = 2 1 ( 1 i )
They are orthogonal since
⟨ ω = e − i θ ∣ ω = e i θ ⟩ = 1 2 ( 1 i ) ( 1 i ) = 1 2 ( 1 + i 2 ) = 0 \langle \omega=\mathrm{e}^{-\mathrm{i}\theta}\mid \omega=\mathrm{e}^{\mathrm{i}\theta}\rangle=\frac{1}{2}(1\quad\mathrm{i})\begin{pmatrix}
1\\\mathrm{i}
\end{pmatrix}=\frac{1}{2}(1+\mathrm{i}^{2})=0 ⟨ ω = e − i θ ∣ ω = e i θ ⟩ = 2 1 ( 1 i ) ( 1 i ) = 2 1 ( 1 + i 2 ) = 0
(4) The matrix of eigenvectors of Ω \Omega Ω is
U = ( 1 2 1 2 − i 2 i 2 ) U=\begin{pmatrix}
\frac{1}{\sqrt{2}} & \frac{1}{\sqrt{2}}\\
-\frac{\mathrm{i}}{\sqrt{2}} & \frac{\mathrm{i}}{\sqrt{2}}
\end{pmatrix} U = ( 2 1 − 2 i 2 1 2 i )
Then
U † Ω U = ( 1 2 i 2 1 2 − i 2 ) ( cos θ sin θ − sin θ cos θ ) ( 1 2 1 2 − i 2 i 2 ) = ( 1 2 ( cos θ − i sin θ ) 1 2 ( sin θ + i cos θ ) 1 2 ( cos θ + i sin θ ) 1 2 ( sin θ − i cos θ ) ) ( 1 2 1 2 − i 2 i 2 ) = ( 1 2 e − i θ i 2 e − i θ 1 2 e i θ − i 2 e i θ ) ( 1 2 1 2 − i 2 i 2 ) = ( e − i θ 0 0 e i θ ) \begin{aligned}
U^{\dagger}\Omega U&=\begin{pmatrix}
\frac{1}{\sqrt{2}} & \frac{\mathrm{i}}{\sqrt{2}}\\
\frac{1}{\sqrt{2}} & -\frac{\mathrm{i}}{\sqrt{2}}
\end{pmatrix}\begin{pmatrix}
\cos \theta & \sin \theta \\
-\sin \theta & \cos \theta
\end{pmatrix}\begin{pmatrix}
\frac{1}{\sqrt{2}} & \frac{1}{\sqrt{2}}\\
-\frac{\mathrm{i}}{\sqrt{2}} & \frac{\mathrm{i}}{\sqrt{2}}
\end{pmatrix}\\
&=\begin{pmatrix}
\frac{1}{\sqrt{2}}(\cos\theta-\mathrm{i}\sin\theta) & \frac{1}{\sqrt{2}}(\sin\theta+\mathrm{i}\cos\theta)\\
\frac{1}{\sqrt{2}}(\cos\theta+\mathrm{i}\sin\theta) & \frac{1}{\sqrt{2}}(\sin\theta-\mathrm{i}\cos\theta)
\end{pmatrix}\begin{pmatrix}
\frac{1}{\sqrt{2}} & \frac{1}{\sqrt{2}}\\
-\frac{\mathrm{i}}{\sqrt{2}} & \frac{\mathrm{i}}{\sqrt{2}}
\end{pmatrix}\\
&=\begin{pmatrix}
\frac{1}{\sqrt{2}}\mathrm{e}^{-\mathrm{i}\theta} & \frac{\mathrm{i}}{\sqrt{2}}\mathrm{e}^{-\mathrm{i}\theta}\\
\frac{1}{\sqrt{2}}\mathrm{e}^{\mathrm{i}\theta} & -\frac{\mathrm{i}}{\sqrt{2}}\mathrm{e}^{\mathrm{i}\theta}
\end{pmatrix}\begin{pmatrix}
\frac{1}{\sqrt{2}} & \frac{1}{\sqrt{2}}\\
-\frac{\mathrm{i}}{\sqrt{2}} & \frac{\mathrm{i}}{\sqrt{2}}
\end{pmatrix}\\
&=\begin{pmatrix}
\mathrm{e}^{-\mathrm{i}\theta} & 0\\
0 & \mathrm{e}^{\mathrm{i}\theta}
\end{pmatrix}
\end{aligned} U † Ω U = ( 2 1 2 1 2 i − 2 i ) ( cos θ − sin θ sin θ cos θ ) ( 2 1 − 2 i 2 1 2 i ) = ( 2 1 ( cos θ − i sin θ ) 2 1 ( cos θ + i sin θ ) 2 1 ( sin θ + i cos θ ) 2 1 ( sin θ − i cos θ ) ) ( 2 1 − 2 i 2 1 2 i ) = ( 2 1 e − i θ 2 1 e i θ 2 i e − i θ − 2 i e i θ ) ( 2 1 − 2 i 2 1 2 i ) = ( e − i θ 0 0 e i θ )
is diagonal.
■ ~\tag*{$\blacksquare$} ■
Exercise 1.8.6 (1) We have seen that the determinant of a matrix is unchanged under a unitary change of basis. Argue now that
det Ω = product of eigenvalues of Ω = ∏ i = 1 n ω i \operatorname{det} \Omega=\text { product of eigenvalues of } \Omega=\prod_{i=1}^n \omega_i det Ω = product of eigenvalues of Ω = i = 1 ∏ n ω i
for a Hermitian or unitary Ω \Omega Ω .
(2) Using the invariance of the trace under the same transformation, show that
Tr Ω = ∑ i = 1 n ω i \operatorname{Tr} \Omega=\sum_{i=1}^n \omega_i Tr Ω = i = 1 ∑ n ω i
Solution. (1) Suppose U U U is the unitary matrix that transforms Ω \Omega Ω into a diagonal matrix D D D with Ω \Omega Ω 's eigenvalues ω i \omega_{i} ω i on its diagonal. Then
det Ω = det ( U † Ω U ) = det D = ∏ i = 1 n ω i \det\Omega=\det(U^{\dagger}\Omega U)=\det D=\prod_{i=1}^{n}\omega_{i} det Ω = det ( U † Ω U ) = det D = i = 1 ∏ n ω i
(2) By using the same transformation, we have
Tr Ω = Tr ( U † Ω U ) = Tr D = ∑ i = 1 n ω i \operatorname{Tr}\Omega=\operatorname{Tr}(U^{\dagger}\Omega U)=\operatorname{Tr} D=\sum_{i=1}^{n}\omega_{i} Tr Ω = Tr ( U † Ω U ) = Tr D = i = 1 ∑ n ω i
■ ~\tag*{$\blacksquare$} ■
Exercise 1.8.7 By using the results on the trace and determinant from the last problem, show that the eigenvalues of the matrix
Ω = ( 1 2 2 1 ) \Omega=\begin{pmatrix}
1 & 2 \\
2 & 1
\end{pmatrix} Ω = ( 1 2 2 1 )
are 3 3 3 and − 1 -1 − 1 . Verify this by explicit computation. Note that the Hermitian nature of the matrix is an essential ingredient.
Solution. According to Exercise 1.8.6, we have
{ ω 1 × ω 2 = det Ω = 1 × 1 − 2 × 2 = − 3 ω 1 + ω 2 = 1 + 1 = 2 \left\{
\begin{aligned}
\omega_{1}\times \omega_{2}&=\det\Omega=1\times 1-2\times 2=-3\\
\omega_{1}+\omega_{2}&=1+1=2
\end{aligned}\right. { ω 1 × ω 2 ω 1 + ω 2 = det Ω = 1 × 1 − 2 × 2 = − 3 = 1 + 1 = 2
Solving the equation, we get
{ ω 1 = − 1 ω 2 = 3 \left\{\begin{aligned}
\omega_{1}&=-1\\
\omega_{2}&=3
\end{aligned}\right. { ω 1 ω 2 = − 1 = 3
For verification, we can calculate the characteristic equation
det ( Ω − ω I ) = ∣ 1 − ω 2 2 1 − ω ∣ = ( 1 − ω ) 2 − 4 = ( 1 − ω + 2 ) ( 1 − ω − 2 ) = 0 \det(\Omega-\omega\mathbb{I})=\left|\begin{matrix}
1-\omega & 2\\
2 & 1-\omega
\end{matrix}\right|=(1-\omega)^{2}-4=(1-\omega+2)(1-\omega-2)=0 det ( Ω − ω I ) = ∣ ∣ 1 − ω 2 2 1 − ω ∣ ∣ = ( 1 − ω ) 2 − 4 = ( 1 − ω + 2 ) ( 1 − ω − 2 ) = 0
We can get the eigenvalues
{ ω 1 = − 1 ω 2 = 3 \left\{\begin{aligned}
\omega_{1}&=-1\\
\omega_{2}&=3
\end{aligned}\right. { ω 1 ω 2 = − 1 = 3
■ ~\tag*{$\blacksquare$} ■
Exercise 1.8.8 Consider Hermitian matrices M 1 , M 2 , M 3 , M 4 M^1, M^2, M^3, M^4 M 1 , M 2 , M 3 , M 4 that obey
M i M j + M j M i = 2 δ i j I , i , j = 1 , … , 4 M^i M^j+M^j M^i=2 \delta^{i j} \mathbb{I}, \quad i, j=1, \ldots, 4 M i M j + M j M i = 2 δ ij I , i , j = 1 , … , 4
(1) Show that the eigenvalues of M i M^i M i are ± 1 \pm 1 ± 1 . (Hint: go to the eigenbasis of M i M^i M i , and use the equation for i = j i=j i = j .)
(2) By considering the relation
M i M i = − M j M i for i ≠ j M^i M^i=-M^j M^i \quad \text { for } i \neq j M i M i = − M j M i for i = j
show that M i M^i M i are traceless. [Hint: Tr ( A C B ) = Tr ( C B A ) \operatorname{Tr}(ACB)=\operatorname{Tr}(CBA) Tr ( A CB ) = Tr ( CB A ) .]
(3) Show that they cannot be odd-dimensional matrices.
Solution. (1) Start with equation
M i M j + M j M i = 2 δ i j I M^{i}M^{j}+M^{j}M^{i}=2\delta^{ij}\mathbb{I} M i M j + M j M i = 2 δ ij I
Take i = j i=j i = j , we get
M i M i = I M^{i}M^{i}=\mathbb{I} M i M i = I
Apply M i M i M^{i}M^{i} M i M i to eigenvector ∣ ω ⟩ |\omega\rangle ∣ ω ⟩ of M i M^{i} M i , we have
M i M i ∣ ω ⟩ = M i ( ω ∣ ω ⟩ ) = ω 2 ∣ ω ⟩ M i M i ∣ ω ⟩ = I ∣ ω ⟩ = ∣ ω ⟩ \begin{aligned}
M^{i}M^{i}|\omega\rangle&=M^{i}(\omega|\omega\rangle)=\omega^{2}|\omega\rangle\\
M^{i}M^{i}|\omega\rangle&=\mathbb{I}|\omega\rangle=|\omega\rangle
\end{aligned} M i M i ∣ ω ⟩ M i M i ∣ ω ⟩ = M i ( ω ∣ ω ⟩) = ω 2 ∣ ω ⟩ = I ∣ ω ⟩ = ∣ ω ⟩
Therefore,
ω 2 = 1 ω = ± 1 \begin{aligned}
\omega^{2}&=1\\
\omega&=\pm 1
\end{aligned} ω 2 ω = 1 = ± 1
(2) From the relation
M i M j = − M j M i M j M i M j = − M j M j M i = − M i \begin{aligned}
M^{i}M^{j}&=-M^{j}M^{i}\\
M^{j}M^{i}M^{j}&=-M^{j}M^{j}M^{i}=-M^{i}
\end{aligned} M i M j M j M i M j = − M j M i = − M j M j M i = − M i
We can take the trace of M i M^{i} M i to get
Tr M i = Tr ( − M j M i M j ) = − Tr ( M j M i M j ) = − Tr ( M i M j M j ) = − Tr ( M i I ) = − Tr ( M i ) = 0 \begin{aligned}
\operatorname{Tr} M^{i}&=\operatorname{Tr}(-M^{j}M^{i}M^{j})\\
&=-\operatorname{Tr}(M^{j}M^{i}M^{j})\\
&=-\operatorname{Tr}(M^{i}M^{j}M^{j})\\
&=-\operatorname{Tr}(M^{i}\mathbb{I})\\
&=-\operatorname{Tr}(M^{i})\\
&=0
\end{aligned} Tr M i = Tr ( − M j M i M j ) = − Tr ( M j M i M j ) = − Tr ( M i M j M j ) = − Tr ( M i I ) = − Tr ( M i ) = 0
M i M^{i} M i is traceless.
(3) According to 1.8.6,
Tr M i = ∑ k = 1 n ω k \operatorname{Tr} M^{i}=\sum_{k=1}^{n}\omega_{k} Tr M i = k = 1 ∑ n ω k
where n n n is the dimension of the matrix. Since ω k = ± 1 \omega_{k}=\pm 1 ω k = ± 1 , Tr M i \operatorname{Tr} M^{i} Tr M i can be zero only if n n n is even. (The sum of an odd number of odd numbers is still odd, and cannot be zero.)
■ ~\tag*{$\blacksquare$} ■
Exercise 1.8.9 A collection of masses m α m_\alpha m α , located at r α \mathbf{r}_\alpha r α and rotating with angular velocity ω \boldsymbol{\omega} ω around a common axis has an angular momentum
l = ∑ α m α ( r α × v α ) \mathbf{l}=\sum_\alpha m_\alpha\left(\mathbf{r}_\alpha \times \mathbf{v}_\alpha\right) l = α ∑ m α ( r α × v α )
where v α = ω × r α \mathbf{v}_\alpha=\boldsymbol{\omega} \times \mathbf{r}_\alpha v α = ω × r α is the velocity of m α m_\alpha m α . By using the identity
A × ( B × C ) = B ( A ⋅ C ) − C ( A ⋅ B ) \mathbf{A} \times(\mathbf{B} \times \mathbf{C})=\mathbf{B}(\mathbf{A} \cdot \mathbf{C})-\mathbf{C}(\mathbf{A} \cdot \mathbf{B}) A × ( B × C ) = B ( A ⋅ C ) − C ( A ⋅ B )
show that each Cartesian component l i l_i l i of l \mathbf{l} l is given by
l i = ∑ j M i j ω j l_i=\sum_j M_{i j} \omega_j l i = j ∑ M ij ω j
where
M i j = ∑ α m α [ r α 2 δ i j − ( r α ) i ( r α ) j ] M_{i j}=\sum_\alpha m_\alpha\left[r_\alpha^2 \delta_{i j}-\left(\mathbf{r}_\alpha\right)_i\left(\mathbf{r}_\alpha\right)_j\right] M ij = α ∑ m α [ r α 2 δ ij − ( r α ) i ( r α ) j ]
or in Dirac notation
∣ l ⟩ = M ∣ ω ⟩ |l\rangle=M|\omega\rangle ∣ l ⟩ = M ∣ ω ⟩
(1) Will the angular momentum and angular velocity always be parallel?
(2) Show that the moment of inertia matrix M i j M_{i j} M ij is Hermitian.
(3) Argue now that there exist three directions for ω \boldsymbol{\omega} ω such that l \mathbf{l} l and ω \boldsymbol{\omega} ω will be parallel. How are these directions to be found?
(4) Consider the moment of inertia matrix of a sphere. Due to the complete symmetry of the sphere, it is clear that every direction is its eigendirection for rotation. What does this say about the three eigenvalues of the matrix M M M ?
Solution. Start from the angular momentum
l = ∑ α m α r α × ( ω × r α ) = ∑ α m α [ ω ( r α ⋅ r α ) − r α ( r α ⋅ ω ) ] = ∑ α m α [ ω r α 2 − r α ( r α ⋅ ω ) ] \begin{aligned}
\mathbf{l}&=\sum_{\alpha} m_{\alpha} \mathbf{r}_{\alpha}\times(\boldsymbol{\omega}\times \mathbf{r}_{\alpha})\\
&=\sum_{\alpha} m_{\alpha} [\boldsymbol{\omega}(\mathbf{r}_{\alpha}\cdot \mathbf{r}_{\alpha})-\mathbf{r}_{\alpha}(\mathbf{r}_{\alpha}\cdot\boldsymbol{\omega})]\\
&=\sum_{\alpha} m_{\alpha} [\boldsymbol{\omega}\,r_{\alpha}^{2}-\mathbf{r}_{\alpha}(\mathbf{r}_{\alpha}\cdot\boldsymbol{\omega})]
\end{aligned} l = α ∑ m α r α × ( ω × r α ) = α ∑ m α [ ω ( r α ⋅ r α ) − r α ( r α ⋅ ω )] = α ∑ m α [ ω r α 2 − r α ( r α ⋅ ω )]
Writing in components, we get
l i = ∑ α m α [ ω i r α 2 − ( r α ) i ( r α ⋅ ω ) ] = ∑ α m α [ ω i r α 2 − ( r α ) i ∑ j ( r α ) j ω j ] = ∑ α m α [ ∑ j δ i j ω j r α 2 − ( r α ) i ∑ j ( r α ) j ω j ] = ∑ j ∑ α m α [ r α 2 δ i j − ( r α ) i ( r α ) j ] ω j ≡ ∑ j M i j ω j \begin{aligned}
l_{i}&=\sum_{\alpha} m_{\alpha}[\omega_{i}\,r_{\alpha}^{2}-(\mathbf{r}_{\alpha})_{i}\,(\mathbf{r}_{\alpha}\cdot\boldsymbol{\omega})]\\
&=\sum_{\alpha} m_{\alpha}[\omega_{i}\,r_{\alpha}^{2}-(\mathbf{r}_{\alpha})_{i}\,\sum_{j}(\mathbf{r}_{\alpha})_{j}\omega_{j}]\\
&=\sum_{\alpha} m_{\alpha}[\sum_{j}\delta_{ij}\omega_{j}\,r_{\alpha}^{2}-(\mathbf{r}_{\alpha})_{i}\,\sum_{j}(\mathbf{r}_{\alpha})_{j}\omega_{j}]\\
&=\sum_{j}\sum_{\alpha} m_{\alpha}[r_{\alpha}^{2}\,\delta_{ij}-(\mathbf{r}_{\alpha})_{i}\,(\mathbf{r}_{\alpha})_{j}]\omega_{j}\\
&\equiv \sum_{j}M_{ij}\omega_{j}
\end{aligned} l i = α ∑ m α [ ω i r α 2 − ( r α ) i ( r α ⋅ ω )] = α ∑ m α [ ω i r α 2 − ( r α ) i j ∑ ( r α ) j ω j ] = α ∑ m α [ j ∑ δ ij ω j r α 2 − ( r α ) i j ∑ ( r α ) j ω j ] = j ∑ α ∑ m α [ r α 2 δ ij − ( r α ) i ( r α ) j ] ω j ≡ j ∑ M ij ω j
where M i j ≡ ∑ α m α [ r α 2 δ i j − ( r α ) i ( r α ) j ] M_{ij}\equiv\sum_{\alpha} m_{\alpha}[r_{\alpha}^{2}\,\delta_{ij}-(\mathbf{r}_{\alpha})_{i}\,(\mathbf{r}_{\alpha})_{j}] M ij ≡ ∑ α m α [ r α 2 δ ij − ( r α ) i ( r α ) j ] . Or in Dirac notation,
∣ l ⟩ = M ∣ ω ⟩ |l\rangle=M|\omega\rangle ∣ l ⟩ = M ∣ ω ⟩
(1) No. The angular momentum and angular velocity are not parallel unless ∣ ω ⟩ |\omega\rangle ∣ ω ⟩ is an eigenvector of $M
(2) M j i ⋆ = ( ∑ α m α [ r α 2 δ j i − ( r α ) j ( r α ) i ] ) ⋆ = ∑ α m α [ r α 2 δ i j − ( r α ) i ( r α ) j ] = M i j M_{ji}^{\star}=(\sum_{\alpha} m_{\alpha}[r_{\alpha}^{2}\,\delta_{ji}-(\mathbf{r}_{\alpha})_{j}\,(\mathbf{r}_{\alpha})_{i}])^{\star}=\sum_{\alpha} m_{\alpha}[r_{\alpha}^{2}\,\delta_{ij}-(\mathbf{r}_{\alpha})_{i}\,(\mathbf{r}_{\alpha})_{j}]=M_{ij} M ji ⋆ = ( ∑ α m α [ r α 2 δ ji − ( r α ) j ( r α ) i ] ) ⋆ = ∑ α m α [ r α 2 δ ij − ( r α ) i ( r α ) j ] = M ij .
(3) Since M M M is Hermitian, we can always find three eigenvectors which are orthogonal to each other by solving the eigen-problem M ∣ ω ⟩ = ω ∣ ω ⟩ M|\omega\rangle=\omega|\omega\rangle M ∣ ω ⟩ = ω ∣ ω ⟩ . And these three eigenvectors denote the three directions for ω \boldsymbol{\omega} ω we want to find in the 3-dimensional Euclidean space.
(4) The complete symmetry of sphere means all directions are equivalent eigendirections. Therefore, the eigenvalues are degenerate.
■ ~\tag*{$\blacksquare$} ■
Exercise 1.8.10 By considering the commutator, show that the following Hermitian matrices may be simultaneously diagonalized. Find the eigenvectors common to both and verify that under a unitary transformation to this basis, both matrices are diagonalized.
Ω = ( 1 0 1 0 0 0 1 0 1 ) , Λ = ( 2 1 1 1 0 − 1 1 − 1 2 ) \Omega=\begin{pmatrix}
1 & 0 & 1 \\
0 & 0 & 0 \\
1 & 0 & 1
\end{pmatrix}, \quad \Lambda=\begin{pmatrix}
2 & 1 & 1 \\
1 & 0 & -1 \\
1 & -1 & 2
\end{pmatrix} Ω = ⎝ ⎛ 1 0 1 0 0 0 1 0 1 ⎠ ⎞ , Λ = ⎝ ⎛ 2 1 1 1 0 − 1 1 − 1 2 ⎠ ⎞
Since Ω \Omega Ω is degenerate and Λ \Lambda Λ is not, you must be prudent in deciding which matrix dictates the choice of basis.
Solution. Since the two Hermitian matrices commute
[ Ω , Λ ] = Ω Λ − Λ Ω = ( 1 0 1 0 0 0 1 0 1 ) ( 2 1 1 1 0 − 1 1 − 1 2 ) − ( 2 1 1 1 0 − 1 1 − 1 2 ) ( 1 0 1 0 0 0 1 0 1 ) = ( 3 0 3 0 0 0 3 0 3 ) − ( 3 0 3 0 0 0 3 0 3 ) = 0 \begin{aligned}
\left[\Omega,\Lambda\right]&=\Omega\Lambda-\Lambda\Omega\\
&=\begin{pmatrix}
1 & 0 & 1 \\
0 & 0 & 0 \\
1 & 0 & 1
\end{pmatrix}\begin{pmatrix}
2 & 1 & 1 \\
1 & 0 & -1 \\
1 & -1 & 2
\end{pmatrix}-\begin{pmatrix}
2 & 1 & 1 \\
1 & 0 & -1 \\
1 & -1 & 2
\end{pmatrix}\begin{pmatrix}
1 & 0 & 1 \\
0 & 0 & 0 \\
1 & 0 & 1
\end{pmatrix}\\
&=\begin{pmatrix}
3 & 0 & 3 \\
0 & 0 & 0 \\
3 & 0 & 3
\end{pmatrix}-\begin{pmatrix}
3 & 0 & 3 \\
0 & 0 & 0 \\
3 & 0 & 3
\end{pmatrix}\\
&=0
\end{aligned} [ Ω , Λ ] = ΩΛ − ΛΩ = ⎝ ⎛ 1 0 1 0 0 0 1 0 1 ⎠ ⎞ ⎝ ⎛ 2 1 1 1 0 − 1 1 − 1 2 ⎠ ⎞ − ⎝ ⎛ 2 1 1 1 0 − 1 1 − 1 2 ⎠ ⎞ ⎝ ⎛ 1 0 1 0 0 0 1 0 1 ⎠ ⎞ = ⎝ ⎛ 3 0 3 0 0 0 3 0 3 ⎠ ⎞ − ⎝ ⎛ 3 0 3 0 0 0 3 0 3 ⎠ ⎞ = 0
They can be diagonalized simultaneously. We choose Λ \Lambda Λ 's characteristic equation
det ( Λ − λ I ) = ∣ 2 − λ 1 1 1 − λ − 1 1 − 1 2 − λ ∣ = ( λ + 1 ) ( 2 − λ ) ( λ − 3 ) = 0 \det(\Lambda-\lambda\mathbb{I})=\left|
\begin{matrix}
2-\lambda & 1 & 1\\
1 & -\lambda & -1\\
1 & -1 & 2-\lambda
\end{matrix}
\right|=(\lambda+1)(2-\lambda)(\lambda-3)=0 det ( Λ − λ I ) = ∣ ∣ 2 − λ 1 1 1 − λ − 1 1 − 1 2 − λ ∣ ∣ = ( λ + 1 ) ( 2 − λ ) ( λ − 3 ) = 0
The eigenvalues are
λ = − 1 , 2 , 3 \lambda=-1, 2, 3 λ = − 1 , 2 , 3
Then the eigenvectors corresponding the eigenvalues are
λ = − 1 : ( 3 1 1 1 1 − 1 1 − 1 3 ) ( x 1 x 2 x 3 ) = 0 ⇒ ∣ λ = − 1 ⟩ = 1 6 ( 1 − 2 − 1 ) λ = 2 : ( 0 1 1 1 − 2 − 1 1 − 1 0 ) ( x 1 x 2 x 3 ) = 0 ⇒ ∣ λ = 2 ⟩ = 1 3 ( 1 1 − 1 ) λ = 3 : ( − 1 1 1 1 − 3 − 1 1 − 1 − 1 ) ( x 1 x 2 x 3 ) = 0 ⇒ ∣ λ = 3 ⟩ = 1 2 ( 1 0 1 ) \begin{aligned}
&\lambda=-1: &~\begin{pmatrix}
3&1&1\\
1&1&-1\\
1&-1&3
\end{pmatrix}\begin{pmatrix}
x_{1}\\x_{2}\\x_{3}
\end{pmatrix}=0\quad &\Rightarrow \quad |\lambda=-1\rangle=\frac{1}{\sqrt{6}}\begin{pmatrix}
1\\-2\\-1
\end{pmatrix}\\
&\lambda=2: &~\begin{pmatrix}
0&1&1\\
1&-2&-1\\
1&-1&0
\end{pmatrix}\begin{pmatrix}
x_{1}\\x_{2}\\x_{3}
\end{pmatrix}=0\quad &\Rightarrow \quad |\lambda=2\rangle=\frac{1}{\sqrt{3}}\begin{pmatrix}
1\\1\\-1
\end{pmatrix}\\
&\lambda=3: &~\begin{pmatrix}
-1&1&1\\
1&-3&-1\\
1&-1&-1
\end{pmatrix}\begin{pmatrix}
x_{1}\\x_{2}\\x_{3}
\end{pmatrix}=0\quad &\Rightarrow \quad |\lambda=3\rangle=\frac{1}{\sqrt{2}}\begin{pmatrix}
1\\0\\1
\end{pmatrix}
\end{aligned} λ = − 1 : λ = 2 : λ = 3 : ⎝ ⎛ 3 1 1 1 1 − 1 1 − 1 3 ⎠ ⎞ ⎝ ⎛ x 1 x 2 x 3 ⎠ ⎞ = 0 ⎝ ⎛ 0 1 1 1 − 2 − 1 1 − 1 0 ⎠ ⎞ ⎝ ⎛ x 1 x 2 x 3 ⎠ ⎞ = 0 ⎝ ⎛ − 1 1 1 1 − 3 − 1 1 − 1 − 1 ⎠ ⎞ ⎝ ⎛ x 1 x 2 x 3 ⎠ ⎞ = 0 ⇒ ∣ λ = − 1 ⟩ = 6 1 ⎝ ⎛ 1 − 2 − 1 ⎠ ⎞ ⇒ ∣ λ = 2 ⟩ = 3 1 ⎝ ⎛ 1 1 − 1 ⎠ ⎞ ⇒ ∣ λ = 3 ⟩ = 2 1 ⎝ ⎛ 1 0 1 ⎠ ⎞
Then the matrix of eigenvectors of Λ \Lambda Λ is
U = ( 1 6 1 3 1 2 − 2 6 1 3 0 − 1 6 − 1 3 1 2 ) U=\begin{pmatrix}
\frac{1}{\sqrt{6}}&\frac{1}{\sqrt{3}}&\frac{1}{\sqrt{2}}\\
-\frac{2}{\sqrt{6}}&\frac{1}{\sqrt{3}}&0\\
-\frac{1}{\sqrt{6}}&-\frac{1}{\sqrt{3}}&\frac{1}{\sqrt{2}}
\end{pmatrix} U = ⎝ ⎛ 6 1 − 6 2 − 6 1 3 1 3 1 − 3 1 2 1 0 2 1 ⎠ ⎞
To verify Ω \Omega Ω and Λ \Lambda Λ are simultanelously diagonalized:
U † Ω U = ( 1 6 − 2 6 − 1 6 1 3 1 3 − 1 3 1 2 0 1 2 ) ( 1 0 1 0 0 0 1 0 1 ) ( 1 6 1 3 1 2 − 2 6 1 3 0 − 1 6 − 1 3 1 2 ) = ( 0 0 0 0 0 0 2 0 2 ) ( 1 6 1 3 1 2 − 2 6 1 3 0 − 1 6 − 1 3 1 2 ) = ( 0 0 0 0 0 0 0 0 2 ) \begin{aligned}
U^{\dagger}\Omega U&=\begin{pmatrix}
\frac{1}{\sqrt{6}}&-\frac{2}{\sqrt{6}}&-\frac{1}{\sqrt{6}}\\
\frac{1}{\sqrt{3}}&\frac{1}{\sqrt{3}}&-\frac{1}{\sqrt{3}}\\
\frac{1}{\sqrt{2}}&0&\frac{1}{\sqrt{2}}
\end{pmatrix}\begin{pmatrix}
1 & 0 & 1 \\
0 & 0 & 0 \\
1 & 0 & 1
\end{pmatrix}\begin{pmatrix}
\frac{1}{\sqrt{6}}&\frac{1}{\sqrt{3}}&\frac{1}{\sqrt{2}}\\
-\frac{2}{\sqrt{6}}&\frac{1}{\sqrt{3}}&0\\
-\frac{1}{\sqrt{6}}&-\frac{1}{\sqrt{3}}&\frac{1}{\sqrt{2}}
\end{pmatrix}\\
&=\begin{pmatrix}
0&0&0\\
0&0&0\\
\sqrt{2}&0&\sqrt{2}
\end{pmatrix}\begin{pmatrix}
\frac{1}{\sqrt{6}}&\frac{1}{\sqrt{3}}&\frac{1}{\sqrt{2}}\\
-\frac{2}{\sqrt{6}}&\frac{1}{\sqrt{3}}&0\\
-\frac{1}{\sqrt{6}}&-\frac{1}{\sqrt{3}}&\frac{1}{\sqrt{2}}
\end{pmatrix}\\
&=\begin{pmatrix}
0 & 0 & 0 \\
0 & 0 & 0 \\
0 & 0 & 2
\end{pmatrix}
\end{aligned} U † Ω U = ⎝ ⎛ 6 1 3 1 2 1 − 6 2 3 1 0 − 6 1 − 3 1 2 1 ⎠ ⎞ ⎝ ⎛ 1 0 1 0 0 0 1 0 1 ⎠ ⎞ ⎝ ⎛ 6 1 − 6 2 − 6 1 3 1 3 1 − 3 1 2 1 0 2 1 ⎠ ⎞ = ⎝ ⎛ 0 0 2 0 0 0 0 0 2 ⎠ ⎞ ⎝ ⎛ 6 1 − 6 2 − 6 1 3 1 3 1 − 3 1 2 1 0 2 1 ⎠ ⎞ = ⎝ ⎛ 0 0 0 0 0 0 0 0 2 ⎠ ⎞
U † Λ U = ( 1 6 − 2 6 − 1 6 1 3 1 3 − 1 3 1 2 0 1 2 ) ( 2 1 1 1 0 − 1 1 − 1 2 ) ( 1 6 1 3 1 2 − 2 6 1 3 0 − 1 6 − 1 3 1 2 ) = ( − 1 6 2 6 1 6 2 3 2 3 − 2 3 3 2 0 3 2 ) ( 1 6 1 3 1 2 − 2 6 1 3 0 − 1 6 − 1 3 1 2 ) = ( − 1 0 0 0 2 0 0 0 3 ) \begin{aligned}
U^{\dagger}\Lambda U&=\begin{pmatrix}
\frac{1}{\sqrt{6}}&-\frac{2}{\sqrt{6}}&-\frac{1}{\sqrt{6}}\\
\frac{1}{\sqrt{3}}&\frac{1}{\sqrt{3}}&-\frac{1}{\sqrt{3}}\\
\frac{1}{\sqrt{2}}&0&\frac{1}{\sqrt{2}}
\end{pmatrix}\begin{pmatrix}
2 & 1 & 1 \\
1 & 0 & -1 \\
1 & -1 & 2
\end{pmatrix}\begin{pmatrix}
\frac{1}{\sqrt{6}}&\frac{1}{\sqrt{3}}&\frac{1}{\sqrt{2}}\\
-\frac{2}{\sqrt{6}}&\frac{1}{\sqrt{3}}&0\\
-\frac{1}{\sqrt{6}}&-\frac{1}{\sqrt{3}}&\frac{1}{\sqrt{2}}
\end{pmatrix}\\
&=\begin{pmatrix}
-\frac{1}{\sqrt{6}}&\frac{2}{\sqrt{6}}&\frac{1}{\sqrt{6}}\\
\frac{2}{\sqrt{3}}&\frac{2}{\sqrt{3}}&-\frac{2}{\sqrt{3}}\\
\frac{3}{\sqrt{2}}&0&\frac{3}{\sqrt{2}}
\end{pmatrix}\begin{pmatrix}
\frac{1}{\sqrt{6}}&\frac{1}{\sqrt{3}}&\frac{1}{\sqrt{2}}\\
-\frac{2}{\sqrt{6}}&\frac{1}{\sqrt{3}}&0\\
-\frac{1}{\sqrt{6}}&-\frac{1}{\sqrt{3}}&\frac{1}{\sqrt{2}}
\end{pmatrix}\\
&=\begin{pmatrix}
-1 & 0 & 0 \\
0 & 2 & 0 \\
0 & 0 & 3
\end{pmatrix}
\end{aligned} U † Λ U = ⎝ ⎛ 6 1 3 1 2 1 − 6 2 3 1 0 − 6 1 − 3 1 2 1 ⎠ ⎞ ⎝ ⎛ 2 1 1 1 0 − 1 1 − 1 2 ⎠ ⎞ ⎝ ⎛ 6 1 − 6 2 − 6 1 3 1 3 1 − 3 1 2 1 0 2 1 ⎠ ⎞ = ⎝ ⎛ − 6 1 3 2 2 3 6 2 3 2 0 6 1 − 3 2 2 3 ⎠ ⎞ ⎝ ⎛ 6 1 − 6 2 − 6 1 3 1 3 1 − 3 1 2 1 0 2 1 ⎠ ⎞ = ⎝ ⎛ − 1 0 0 0 2 0 0 0 3 ⎠ ⎞
■ ~\tag*{$\blacksquare$} ■
Exercise 1.8.11 Consider the coupled mass problem discussed above.
(1) Given that the initial state is ∣ 1 ⟩ |1\rangle ∣1 ⟩ , in which the first mass is displaced by unity and the second is left alone, calculate ∣ 1 ( t ) ⟩ |1(t)\rangle ∣1 ( t )⟩ by following the algorithm.
(2) Compare your result with that following from Eq. (1.8.39).
Solution.
(1) Equation of motion
d 2 d t 2 ( x 1 x 2 ) = ( − 2 k m k m k m − 2 k m ) ( x 1 x 2 ) \frac{\mathrm{d}^{2}}{\mathrm{d}t^{2}}\begin{pmatrix}
x_{1}\\
x_{2}
\end{pmatrix}=\begin{pmatrix}
-\frac{2k}{m} & \frac{k}{m}\\
\frac{k}{m} & -\frac{2k}{m}
\end{pmatrix}\begin{pmatrix}
x_{1}\\
x_{2}
\end{pmatrix} d t 2 d 2 ( x 1 x 2 ) = ( − m 2 k m k m k − m 2 k ) ( x 1 x 2 )
Set
∣ − 2 k m + ω 2 k m k m − 2 k m + ω 2 ∣ = 0 \left|\begin{matrix}
-\frac{2k}{m}+\omega^{2} & \frac{k}{m}\\
\frac{k}{m} & -\frac{2k}{m}+\omega^{2}
\end{matrix}\right|=0 ∣ ∣ − m 2 k + ω 2 m k m k − m 2 k + ω 2 ∣ ∣ = 0
we have
ω 1 = 3 k m ω 2 = k m \omega_{1}=\sqrt{\frac{3k}{m}}\quad \omega_{2}=\sqrt{\frac{k}{m}} ω 1 = m 3 k ω 2 = m k
The corresponding eigenvectors are
∣ ω 1 ⟩ = 1 2 ( 1 − 1 ) ∣ ω 2 ⟩ = 1 2 ( 1 1 ) |\omega_{1}\rangle=\frac{1}{\sqrt{2}}\begin{pmatrix}
1\\-1
\end{pmatrix}\quad
|\omega_{2}\rangle=\frac{1}{\sqrt{2}}\begin{pmatrix}
1\\1
\end{pmatrix} ∣ ω 1 ⟩ = 2 1 ( 1 − 1 ) ∣ ω 2 ⟩ = 2 1 ( 1 1 )
Then the matrix of eigenvectors is
Λ ≡ ( 1 2 1 2 − 1 2 1 2 ) \Lambda\equiv\begin{pmatrix}
\frac{1}{\sqrt{2}} & \frac{1}{\sqrt{2}}\\
-\frac{1}{\sqrt{2}} & \frac{1}{\sqrt{2}}
\end{pmatrix} Λ ≡ ( 2 1 − 2 1 2 1 2 1 )
It can diagonalize the original matrix
Λ † ( − 2 k m k m k m − 2 k m ) Λ = ( − ω 1 2 0 0 − ω 2 2 ) \Lambda^{\dagger}\begin{pmatrix}
-\frac{2k}{m} & \frac{k}{m}\\
\frac{k}{m} & -\frac{2k}{m}
\end{pmatrix}\Lambda=\begin{pmatrix}
-\omega_{1}^{2} & 0\\
0 & -\omega_{2}^{2}
\end{pmatrix} Λ † ( − m 2 k m k m k − m 2 k ) Λ = ( − ω 1 2 0 0 − ω 2 2 )
In eigenbasis,
( x ¨ I x ¨ I I ) = ( − ω 1 2 0 0 − ω 2 2 ) ( x I x I I ) ⇒ { x I ( t ) = x I ( 0 ) cos ω 1 t x I I ( t ) = x I I ( 0 ) cos ω 2 t ( ⋆ ) \begin{pmatrix}
\ddot{x}_{I}\\
\ddot{x}_{II}
\end{pmatrix}=
\begin{pmatrix}
-\omega_{1}^{2} & 0\\
0 & -\omega_{2}^{2}
\end{pmatrix}
\begin{pmatrix}
x_{I}\\
x_{II}
\end{pmatrix}\quad \Rightarrow\quad
\left\{
\begin{aligned}
x_{I}(t)&=x_{I}(0)\cos\omega_{1}t\\
x_{II}(t)&=x_{II}(0)\cos\omega_{2}t
\end{aligned}\right.\tag{$\star$} ( x ¨ I x ¨ II ) = ( − ω 1 2 0 0 − ω 2 2 ) ( x I x II ) ⇒ { x I ( t ) x II ( t ) = x I ( 0 ) cos ω 1 t = x II ( 0 ) cos ω 2 t ( ⋆ )
In this problem,
∣ 1 ⟩ = ( x 1 ( 0 ) x 2 ( 0 ) ) = ( 1 0 ) |1\rangle=\begin{pmatrix}
x_{1}(0)\\
x_{2}(0)
\end{pmatrix}=\begin{pmatrix}
1\\
0
\end{pmatrix} ∣1 ⟩ = ( x 1 ( 0 ) x 2 ( 0 ) ) = ( 1 0 )
We first transform it into eigenbasis
( x I ( t ) x I I ( t ) ) = Λ † ( x 1 ( 0 ) x 2 ( 0 ) ) = ( 1 2 − 1 2 1 2 1 2 ) ( 1 0 ) = ( 1 2 1 2 ) \begin{pmatrix}
x_{I}(t)\\
x_{II}(t)
\end{pmatrix}=\Lambda^{\dagger}
\begin{pmatrix}
x_{1}(0)\\
x_{2}(0)
\end{pmatrix}=\begin{pmatrix}
\frac{1}{\sqrt{2}} & -\frac{1}{\sqrt{2}}\\
\frac{1}{\sqrt{2}} & \frac{1}{\sqrt{2}}
\end{pmatrix}\begin{pmatrix}
1\\
0
\end{pmatrix}=\begin{pmatrix}
\frac{1}{\sqrt{2}}\\
\frac{1}{\sqrt{2}}
\end{pmatrix} ( x I ( t ) x II ( t ) ) = Λ † ( x 1 ( 0 ) x 2 ( 0 ) ) = ( 2 1 2 1 − 2 1 2 1 ) ( 1 0 ) = ( 2 1 2 1 )
In the eigenbasis, the state evolves according to (⋆ \star ⋆ ).
( x I ( t ) x I I ( t ) ) = ( 1 2 cos ω 1 t 1 2 cos ω 2 t ) \begin{pmatrix}
x_{I}(t)\\
x_{II}(t)
\end{pmatrix}=\begin{pmatrix}
\frac{1}{\sqrt{2}}\cos\omega_{1}t\\
\frac{1}{\sqrt{2}}
\cos\omega_{2}t
\end{pmatrix} ( x I ( t ) x II ( t ) ) = ( 2 1 cos ω 1 t 2 1 cos ω 2 t )
Then we transform it back to the original basis:
∣ 1 ( t ) ⟩ = ( x 1 ( t ) x 2 ( t ) ) = Λ ( x I ( t ) x I I ( t ) ) = ( 1 2 1 2 − 1 2 1 2 ) ( 1 2 cos ω 1 t 1 2 cos ω 2 t ) = ( 1 2 cos 3 k m t + 1 2 cos k m t − 1 2 cos 3 k m t + 1 2 cos k m t ) \begin{aligned}
|1(t)\rangle&=\begin{pmatrix}
x_{1}(t)\\
x_{2}(t)
\end{pmatrix}=\Lambda\begin{pmatrix}
x_{I}(t)\\
x_{II}(t)
\end{pmatrix}\\
&=\begin{pmatrix}
\frac{1}{\sqrt{2}} & \frac{1}{\sqrt{2}}\\
-\frac{1}{\sqrt{2}} & \frac{1}{\sqrt{2}}
\end{pmatrix}\begin{pmatrix}
\frac{1}{\sqrt{2}}\cos\omega_{1}t\\
\frac{1}{\sqrt{2}}\cos\omega_{2}t
\end{pmatrix}\\
&=\begin{pmatrix}
\frac{1}{2}\cos\sqrt{\frac{3k}{m}}t+\frac{1}{2}\cos\sqrt{\frac{k}{m}}t\\
-\frac{1}{2}\cos\sqrt{\frac{3k}{m}}t+\frac{1}{2}\cos\sqrt{\frac{k}{m}}t
\end{pmatrix}
\end{aligned} ∣1 ( t )⟩ = ( x 1 ( t ) x 2 ( t ) ) = Λ ( x I ( t ) x II ( t ) ) = ( 2 1 − 2 1 2 1 2 1 ) ( 2 1 cos ω 1 t 2 1 cos ω 2 t ) = ⎝ ⎛ 2 1 cos m 3 k t + 2 1 cos m k t − 2 1 cos m 3 k t + 2 1 cos m k t ⎠ ⎞
(2) By subsitituting ( x 1 ( 0 ) x 2 ( 0 ) ) = ( 1 0 ) \begin{pmatrix}
x_{1}(0)\\
x_{2}(0)
\end{pmatrix}=\begin{pmatrix}
1\\0
\end{pmatrix} ( x 1 ( 0 ) x 2 ( 0 ) ) = ( 1 0 ) in equation (1.8.39), we can get the same solution:
( x 1 ( t ) x 2 ( t ) ) = ( 1 2 cos 3 k m t + 1 2 cos k m t − 1 2 cos 3 k m t + 1 2 cos k m t ) \begin{pmatrix}
x_{1}(t)\\
x_{2}(t)
\end{pmatrix}=\begin{pmatrix}
\frac{1}{2}\cos\sqrt{\frac{3k}{m}}t+\frac{1}{2}\cos\sqrt{\frac{k}{m}}t\\
-\frac{1}{2}\cos\sqrt{\frac{3k}{m}}t+\frac{1}{2}\cos\sqrt{\frac{k}{m}}t
\end{pmatrix} ( x 1 ( t ) x 2 ( t ) ) = ⎝ ⎛ 2 1 cos m 3 k t + 2 1 cos m k t − 2 1 cos m 3 k t + 2 1 cos m k t ⎠ ⎞
■ ~\tag*{$\blacksquare$} ■
Exercise 1.8.12 Consider once again the problem discussed in the previous example.
(1) Assuming that
∣ x ¨ ⟩ = Ω ∣ x ⟩ |\ddot{x}\rangle=\Omega|x\rangle ∣ x ¨ ⟩ = Ω∣ x ⟩
has a solution
∣ x ( t ) ⟩ = U ( t ) ∣ x ( 0 ) ⟩ |x(t)\rangle=U(t)|x(0)\rangle ∣ x ( t )⟩ = U ( t ) ∣ x ( 0 )⟩
find the differential equation satisfied by U ( t ) U(t) U ( t ) . Use the fact that ∣ x ( 0 ) ⟩ |x(0)\rangle ∣ x ( 0 )⟩ is arbitrary.
(2) Assuming (as is the case) that Ω \Omega Ω and U U U can be simultaneously diagonalized, solve for the elements of the matrix U U U in this common basis and regain Eq. (1.8.43). Assume ∣ x ˙ ( 0 ) ⟩ = 0 |\dot{x}(0)\rangle=0 ∣ x ˙ ( 0 )⟩ = 0 .
Solution. (1) Assuming that
∣ x ¨ ( t ) ⟩ = Ω ∣ x ( t ) ⟩ |\ddot{x}(t)\rangle=\Omega|x(t)\rangle ∣ x ¨ ( t )⟩ = Ω∣ x ( t )⟩
has a solution
∣ x ( t ) ⟩ = U ( t ) ∣ x ( 0 ) ⟩ |x(t)\rangle=U(t)|x(0)\rangle ∣ x ( t )⟩ = U ( t ) ∣ x ( 0 )⟩
Then we can get
d 2 d t 2 U ( t ) ∣ x ( 0 ) ⟩ = Ω U ( t ) ∣ x ( 0 ) ⟩ ( d 2 d t 2 − Ω ) U ( t ) ∣ x ( 0 ) ⟩ = 0 \begin{aligned}
\frac{\mathrm{d}^{2}}{\mathrm{d}t^{2}}U(t)|x(0)\rangle&=\Omega U(t)|x(0)\rangle\\
\left(\frac{\mathrm{d}^{2}}{\mathrm{d}t^{2}}-\Omega\right)U(t)|x(0)\rangle&=0
\end{aligned} d t 2 d 2 U ( t ) ∣ x ( 0 )⟩ ( d t 2 d 2 − Ω ) U ( t ) ∣ x ( 0 )⟩ = Ω U ( t ) ∣ x ( 0 )⟩ = 0
Since ∣ x ( 0 ) ⟩ |x(0)\rangle ∣ x ( 0 )⟩ is arbitrary, we get the differential equation
d 2 d t 2 U ( t ) − Ω U ( t ) = 0 \frac{\mathrm{d}^{2}}{\mathrm{d}t^{2}}U(t)-\Omega U(t)=0 d t 2 d 2 U ( t ) − Ω U ( t ) = 0
(2) From Exercise 1.8.11, we know the Λ = ( 1 2 1 2 − 1 2 1 2 ) \Lambda=\left(\begin{array}{cc}\frac{1}{\sqrt{2}} & \frac{1}{\sqrt{2}} \\ -\frac{1}{\sqrt{2}} & \frac{1}{\sqrt{2}}\end{array}\right) Λ = ( 2 1 − 2 1 2 1 2 1 ) can diagonalize Ω \Omega Ω , and therefore, it can also diagonalize U U U .
In this common basis, we have
( U ¨ 11 ( t ) 0 0 U ¨ 22 ( t ) ) − ( − ω 1 2 0 0 − ω 2 2 ) ( U 11 ( t ) 0 0 U 22 ( t ) ) = 0 \begin{pmatrix}
\ddot{U}_{11}(t) & 0 \\
0 & \ddot{U}_{22}(t)
\end{pmatrix}-\begin{pmatrix}
-\omega_1^2 & 0 \\
0 & -\omega_2^2
\end{pmatrix}\begin{pmatrix}
U_{11}(t) & 0 \\
0 & U_{22}(t)
\end{pmatrix}=0 ( U ¨ 11 ( t ) 0 0 U ¨ 22 ( t ) ) − ( − ω 1 2 0 0 − ω 2 2 ) ( U 11 ( t ) 0 0 U 22 ( t ) ) = 0
⇒ { U ¨ 11 ( t ) + ω 1 2 U 11 ( t ) = 0 U ¨ 22 ( t ) + ω 2 2 U 22 ( t ) = 0 \Rightarrow\quad\left\{\begin{aligned}
\ddot{U}_{11}(t)+\omega_1^2 U_{11}(t)=0 \\
\ddot{U}_{22}(t)+\omega_2^2 U_{22}(t)=0
\end{aligned}\right. ⇒ { U ¨ 11 ( t ) + ω 1 2 U 11 ( t ) = 0 U ¨ 22 ( t ) + ω 2 2 U 22 ( t ) = 0
⇒ { U 11 = A 1 cos ω 1 t + B 1 sin ω 1 t U 22 = A 2 cos ω 2 t + B 2 sin ω 2 t \Rightarrow\quad\left\{\begin{aligned}
U_{11}=A_1 \cos \omega_1 t+B_1 \sin \omega_1 t \\
U_{22}=A_2 \cos \omega_2 t+B_2 \sin \omega_2 t
\end{aligned}\right. ⇒ { U 11 = A 1 cos ω 1 t + B 1 sin ω 1 t U 22 = A 2 cos ω 2 t + B 2 sin ω 2 t
Then
∣ x ˙ ( 0 ) ⟩ = d d t [ U ( t ) ∣ x ( 0 ) ⟩ ] ∣ t = 0 = ( d d t U ( t ) ) ∣ t = 0 ∣ x ( 0 ) ⟩ = 0 |\dot{x}(0)\rangle=\left.\frac{\mathrm{d}}{\mathrm{d} t}[U(t)|x(0)\rangle]\right|_{t=0}=\left.\left(\frac{\mathrm{d}}{\mathrm{d} t} U(t)\right)\right|_{t=0}|x(0)\rangle=0 ∣ x ˙ ( 0 )⟩ = d t d [ U ( t ) ∣ x ( 0 )⟩] ∣ ∣ t = 0 = ( d t d U ( t ) ) ∣ ∣ t = 0 ∣ x ( 0 )⟩ = 0
Since ∣ x ( 0 ) ⟩ |x(0)\rangle ∣ x ( 0 )⟩ is arbitrary, we have
d d t U ( t ) ∣ t = 0 = 0 \left.\frac{\mathrm{d}}{\mathrm{d} t} U(t)\right|_{t=0}=0 d t d U ( t ) ∣ ∣ t = 0 = 0
which means
U ˙ 11 ( 0 ) = U ˙ 22 ( 0 ) = 0 B 1 = B 2 = 0 \begin{aligned}
\dot{U}_{11}(0)&=\dot{U}_{22}(0)=0\\
B_1&=B_2=0
\end{aligned} U ˙ 11 ( 0 ) B 1 = U ˙ 22 ( 0 ) = 0 = B 2 = 0
To satisfy that U U U is unitary, we have
A 1 = A 2 = 1 A_{1}=A_{2}=1 A 1 = A 2 = 1
Therefore,
U = ( cos ω 1 t 0 0 cos ω 2 t ) U=\begin{pmatrix}
\cos\omega_{1}t & 0\\
0 & \cos\omega_{2}t
\end{pmatrix} U = ( cos ω 1 t 0 0 cos ω 2 t )
which is the same as equation (1.8.43).
■ ~\tag*{$\blacksquare$} ■
1.9 Functions of Operators and Related Concepts
Exercise 1.9.1 We know that the series
f ( x ) = ∑ n = 0 ∞ x n f(x)=\sum_{n=0}^{\infty} x^n f ( x ) = n = 0 ∑ ∞ x n
may be equated to the function f ( x ) = ( 1 − x ) − 1 f(x)=(1-x)^{-1} f ( x ) = ( 1 − x ) − 1 if ∣ x ∣ < 1 |x|<1 ∣ x ∣ < 1 . By going to the eigenbasis, examine when the q q q number power series
f ( Ω ) = ∑ n = 0 ∞ Ω n f(\Omega)=\sum_{n=0}^{\infty} \Omega^n f ( Ω ) = n = 0 ∑ ∞ Ω n
of a Hermitian operator Ω \Omega Ω may be identified with ( 1 − Ω ) − 1 (1-\Omega)^{-1} ( 1 − Ω ) − 1 .
Solution. In the eigenbasis,
Ω = ( ω 1 ω 2 ⋱ ω m ) \Omega=\begin{pmatrix}
\omega_{1} & & &\\
&\omega_{2}& &\\
& & \ddots &\\
& & &\omega_{m}
\end{pmatrix} Ω = ⎝ ⎛ ω 1 ω 2 ⋱ ω m ⎠ ⎞
where ω i \omega_{i} ω i are eigenvalues.
f ( Ω ) = ∑ n = 0 ∞ Ω n = ( ∑ n = 0 ∞ ω 1 n ⋱ ∑ n = 0 ∞ ω m n ) = ( 1 1 − ω 1 ⋱ 1 1 − ω m ) = 1 1 − Ω f(\Omega)=\sum_{n=0}^{\infty}\Omega^{n}=\begin{pmatrix}
\sum\limits_{n=0}^{\infty}\omega_{1}^{n} & &\\
& \ddots &\\
& &\sum\limits_{n=0}^{\infty}\omega_{m}^{n}
\end{pmatrix}=\begin{pmatrix}
\frac{1}{1-\omega_{1}} & &\\
& \ddots &\\
& &\frac{1}{1-\omega_{m}}
\end{pmatrix}=\frac{1}{1-\Omega} f ( Ω ) = n = 0 ∑ ∞ Ω n = ⎝ ⎛ n = 0 ∑ ∞ ω 1 n ⋱ n = 0 ∑ ∞ ω m n ⎠ ⎞ = ⎝ ⎛ 1 − ω 1 1 ⋱ 1 − ω m 1 ⎠ ⎞ = 1 − Ω 1
The third equality holds if and only if ∣ ω i ∣ < 1 |\omega_{i}|<1 ∣ ω i ∣ < 1 for i = 1 , … , m i=1,\ldots,m i = 1 , … , m . Therefore, f ( Ω ) f(\Omega) f ( Ω ) can be defined as 1 1 − Ω \frac{1}{1-\Omega} 1 − Ω 1 if and only if the absolute value of each of Ω \Omega Ω 's eigenvalues is less than 1 1 1 .
■ ~\tag*{$\blacksquare$} ■
Exercise 1.9.2 If H H H is a Hermitian operator, show that U = e i H U=\mathrm{e}^{\mathrm{i} H} U = e i H is unitary. (Notice the analogy with c c c numbers: if θ \theta θ is real, u = e i θ u=\mathrm{e}^{\mathrm{i} \theta} u = e i θ is a number of unit modulus.)
Solution. Since H H H is Hermitian, it satisfies
We can compute
U † = ( e i H ) † = e − i H † = e − i H U^{\dagger}=(\mathrm{e}^{\mathrm{i}H})^{\dagger}=\mathrm{e}^{-\mathrm{i}H^{\dagger}}=\mathrm{e}^{-\mathrm{i}H} U † = ( e i H ) † = e − i H † = e − i H
Then(The second equality holds only for commuting operators.)
U † U = e − i H e i H = e − i H + i H = 1 U^{\dagger}U=\mathrm{e}^{-\mathrm{i}H}\mathrm{e}^{\mathrm{i}H}=\mathrm{e}^{-\mathrm{i}H+\mathrm{i}H}=1 U † U = e − i H e i H = e − i H + i H = 1
Therefore U U U is unitary.
■ ~\tag*{$\blacksquare$} ■
Exercise 1.9.3 For the case above, show that det U = e i Tr H \det U=\mathrm{e}^{\mathrm{i}\operatorname{Tr} H} det U = e i Tr H .
Solution. In the eigenbasis of H H H ,
U = e i H = ( ∑ n = 0 ∞ ( i ϵ 1 ) n n ! ⋱ ∑ n = 0 ∞ ( i ϵ m ) n n ! ) = ( e i ϵ 1 ⋱ e i ϵ m ) U=\mathrm{e}^{\mathrm{i}H}=\begin{pmatrix}
\sum\limits_{n=0}^{\infty}\frac{(\mathrm{i}\epsilon_{1})^{n}}{n!} & &\\
& \ddots &\\
& &\sum\limits_{n=0}^{\infty}\frac{(\mathrm{i}\epsilon_{m})^{n}}{n!}
\end{pmatrix}=\begin{pmatrix}
\mathrm{e}^{\mathrm{i}\epsilon_{1}}&\\
& \ddots &\\
& &\mathrm{e}^{\mathrm{i}\epsilon_{m}}
\end{pmatrix} U = e i H = ⎝ ⎛ n = 0 ∑ ∞ n ! ( i ϵ 1 ) n ⋱ n = 0 ∑ ∞ n ! ( i ϵ m ) n ⎠ ⎞ = ⎝ ⎛ e i ϵ 1 ⋱ e i ϵ m ⎠ ⎞
where ϵ 1 , … , ϵ m \epsilon_{1},\ldots,\epsilon_{m} ϵ 1 , … , ϵ m are eigenvalues of H H H , i.e.
H = ( ϵ 1 ⋱ ϵ m ) H=\begin{pmatrix}
\epsilon_{1}&\\
& \ddots &\\
& &\epsilon_{m}
\end{pmatrix} H = ⎝ ⎛ ϵ 1 ⋱ ϵ m ⎠ ⎞
Therefore,
det U = ∏ i = 1 m e i ϵ i = e i ∑ i = 1 m ϵ i = e i Tr H \det U=\prod_{i=1}^{m}\mathrm{e}^{\mathrm{i}\epsilon_{i}}=\mathrm{e}^{\mathrm{i}\sum\limits_{i=1}^{m}\epsilon_{i}}=\mathrm{e}^{\mathrm{i}\operatorname{Tr} H} det U = i = 1 ∏ m e i ϵ i = e i i = 1 ∑ m ϵ i = e i Tr H
■ ~\tag*{$\blacksquare$} ■
1.10 Generalization to Infinite Dimensions
Exercise 1.10.1 Show that δ ( a x ) = δ ( x ) / ∣ a ∣ \delta(a x)=\delta(x) /|a| δ ( a x ) = δ ( x ) /∣ a ∣ . [Consider ∫ δ ( a x ) d ( a x ) \int \delta(a x)\,\mathrm{d}(a x) ∫ δ ( a x ) d ( a x ) . Remember that δ ( x ) = δ ( − x ) \delta(x)=\delta(-x) δ ( x ) = δ ( − x ) .]
Solution. Since δ ( x ) = δ ( − x ) \delta(x)=\delta(-x) δ ( x ) = δ ( − x ) , we have
δ ( a x ) = δ ( ∣ a ∣ x ) \delta(a x)=\delta(|a|x) δ ( a x ) = δ ( ∣ a ∣ x )
Therefore,
∫ − ∞ ∞ δ ( a x ) d x = ∫ − ∞ ∞ δ ( ∣ a ∣ x ) d x = ∫ − ∞ ∞ δ ( ∣ a ∣ x ) ⋅ 1 ∣ a ∣ d ( ∣ a ∣ x ) = 1 ∣ a ∣ ∫ − ∞ ∞ δ ( ∣ a ∣ x ) d ( ∣ a ∣ x ) = 1 ∣ a ∣ ∫ − ∞ ∞ δ ( x ) d x ( change ∣ a ∣ x to x ) \begin{aligned}
\int_{-\infty}^{\infty} \delta(a x) d x & =\int_{-\infty}^{\infty} \delta(|a| x) d x=\int_{-\infty}^{\infty} \delta(|a| x) \cdot \frac{1}{|a|} \mathrm{d}(|a| x) \\
&=\frac{1}{|a|} \int_{-\infty}^{\infty} \delta(|a| x)\,\mathrm{d}(|a| x) \quad \tag{change $|a|x$ to $x$} \\
& =\frac{1}{|a|} \int_{-\infty}^{\infty} \delta(x) \,\mathrm{d} x
\end{aligned} ∫ − ∞ ∞ δ ( a x ) d x = ∫ − ∞ ∞ δ ( ∣ a ∣ x ) d x = ∫ − ∞ ∞ δ ( ∣ a ∣ x ) ⋅ ∣ a ∣ 1 d ( ∣ a ∣ x ) = ∣ a ∣ 1 ∫ − ∞ ∞ δ ( ∣ a ∣ x ) d ( ∣ a ∣ x ) = ∣ a ∣ 1 ∫ − ∞ ∞ δ ( x ) d x ( change ∣ a ∣ x to x )
Thus,
δ ( a x ) = δ ( x ) / ∣ a ∣ \delta(ax)=\delta(x)/|a| δ ( a x ) = δ ( x ) /∣ a ∣
■ ~\tag*{$\blacksquare$} ■
Exercise 1.10.2 Show that
δ ( f ( x ) ) = ∑ i δ ( x i − x ) ∣ d f / d x i ∣ \delta(f(x))=\sum_i \frac{\delta\left(x_i-x\right)}{\left|\mathrm{d} f / \mathrm{d} x_i\right|} δ ( f ( x )) = i ∑ ∣ d f / d x i ∣ δ ( x i − x )
where x i x_i x i are the zeros of f ( x ) f(x) f ( x ) . Hint: Where does δ ( f ( x ) ) \delta(f(x)) δ ( f ( x )) blow up? Expand f ( x ) f(x) f ( x ) near such points in a Taylor series, keeping the first nonzero term.
Solution. Expand f ( x ) f(x) f ( x ) around x i x_{i} x i , where f ( x i ) = 0 f(x_{i})=0 f ( x i ) = 0 :
f ( x ) = f ( x i ) + f ′ ( x i ) ( x − x i ) + 1 2 f ′ ′ ( x i ) ( x − x i ) 2 + ⋯ = 0 + f ′ ( x i ) ( x − x i ) + O [ ( x − x i ) 2 ] ≈ f ′ ( x i ) ( x − x i ) \begin{aligned}
f(x)&=f(x_{i})+f^{\prime}(x_{i})(x-x_{i})+\frac{1}{2}f^{\prime\prime}(x_{i})(x-x_{i})^{2}+\cdots\\
&=0+f^{\prime}(x_{i})(x-x_{i})+\mathcal{O}[(x-x_{i})^{2}]\\
&\approx f^{\prime}(x_{i})(x-x_{i})
\end{aligned} f ( x ) = f ( x i ) + f ′ ( x i ) ( x − x i ) + 2 1 f ′′ ( x i ) ( x − x i ) 2 + ⋯ = 0 + f ′ ( x i ) ( x − x i ) + O [( x − x i ) 2 ] ≈ f ′ ( x i ) ( x − x i )
Introduce a test function g ( x ) g(x) g ( x ) ,\footnote{We use the Exercise 1.10.1 at the last equality.}
∫ − ∞ ∞ g ( x ) δ ( f ( x ) ) d x = ∑ i ∫ x i − ϵ x i + ϵ g ( x ) δ ( f ( x ) ) d x = ∑ i ∫ x i − ϵ x i + ϵ g ( x ) δ ( d f d x ∣ x = x i ( x − x i ) ) d x = ∑ i ∫ x i − ϵ x i + ϵ g ( x ) δ ( x − x i ) ∣ d f d x ∣ x = x i ∣ d x \begin{aligned}
\int_{-\infty}^{\infty}g(x)\delta(f(x))\mathrm{d}x&=\sum_{i}\int_{x_{i}-\epsilon}^{x_{i}+\epsilon}g(x)\,\delta(f(x))\,\mathrm{d}x\\
&=\sum_{i}\int_{x_{i}-\epsilon}^{x_{i}+\epsilon} g(x)\, \delta\left(\frac{\mathrm{d}f}{\mathrm{d}x}\bigg|_{x=x_{i}}(x-x_{i})\right)\mathrm{d}x\\
&=\sum_{i}\int_{x_{i}-\epsilon}^{x_{i}+\epsilon} g(x)\, \frac{\delta(x-x_{i})}{\left|\frac{\mathrm{d}f}{\mathrm{d}x}\bigg|_{x=x_{i}}\right|}\mathrm{d}x
\end{aligned} ∫ − ∞ ∞ g ( x ) δ ( f ( x )) d x = i ∑ ∫ x i − ϵ x i + ϵ g ( x ) δ ( f ( x )) d x = i ∑ ∫ x i − ϵ x i + ϵ g ( x ) δ ( d x d f ∣ ∣ x = x i ( x − x i ) ) d x = i ∑ ∫ x i − ϵ x i + ϵ g ( x ) ∣ ∣ d x d f ∣ ∣ x = x i ∣ ∣ δ ( x − x i ) d x
Therefore,
δ ( f ( x ) ) = ∑ i δ ( x − x i ) ∣ f ′ ( x i ) ∣ \delta(f(x))=\sum_{i}\frac{\delta(x-x_{i})}{|f^{\prime}(x_{i})|} δ ( f ( x )) = i ∑ ∣ f ′ ( x i ) ∣ δ ( x − x i )
■ ~\tag*{$\blacksquare$} ■
Exercise 1.10.3 Consider the theta function θ ( x − x ′ ) \theta(x-x^{\prime}) θ ( x − x ′ ) which vanishes if x − x ′ x-x^{\prime} x − x ′ is negative and equals 1 if x − x ′ x-x^{\prime} x − x ′ is positive. Show that δ ( x − x ′ ) = d d x θ ( x − x ′ ) \delta(x-x^{\prime})=\frac{\mathrm{d}}{\mathrm{d} x} \theta(x-x^{\prime}) δ ( x − x ′ ) = d x d θ ( x − x ′ ) .
Solution. Introduce a test function\footnote{g ( − ∞ ) g(-\infty) g ( − ∞ ) and g ( ∞ ) g(\infty) g ( ∞ ) are finite} g ( x ) g(x) g ( x ) , we have
∫ − ∞ ∞ g ( x ) d d x θ ( x − x ′ ) d x = ∫ − ∞ ∞ d θ ( x − x ′ ) = θ ( x − x ′ ) g ( x ) ∣ − ∞ ∞ − ∫ − ∞ ∞ θ ( x − x ′ ) g ′ ( x ) d x = 1 ⋅ g ( ∞ ) − 0 ⋅ g ( − ∞ ) − ∞ 0 ∞ g ′ ( x ) d x = g ( ∞ ) − [ g ( ∞ ) − g ( 0 ) ] = g ( 0 ) = ∫ − ∞ ∞ g ( x ) δ ( x ) d x \begin{aligned}
\int_{-\infty}^{\infty}g(x)\,\frac{\mathrm{d}}{\mathrm{d}x}\theta(x-x^{\prime})\mathrm{d}x&=\int_{-\infty}^{\infty}\mathrm{d}\theta(x-x^{\prime})\\
&=\theta(x-x^{\prime})g(x)\bigg|_{-\infty}^{\infty}-\int_{-\infty}^{\infty}\theta(x-x^{\prime})g^{\prime}(x)\mathrm{d}x\\
&=1\cdot g(\infty)-0\cdot g(-\infty)-\infty_{0}^{\infty}g^{\prime}(x)\mathrm{d}x\\
&=g(\infty)-[g(\infty)-g(0)]\\
&=g(0)\\
&=\int_{-\infty}^{\infty}g(x)\delta(x)\mathrm{d}x
\end{aligned} ∫ − ∞ ∞ g ( x ) d x d θ ( x − x ′ ) d x = ∫ − ∞ ∞ d θ ( x − x ′ ) = θ ( x − x ′ ) g ( x ) ∣ ∣ − ∞ ∞ − ∫ − ∞ ∞ θ ( x − x ′ ) g ′ ( x ) d x = 1 ⋅ g ( ∞ ) − 0 ⋅ g ( − ∞ ) − ∞ 0 ∞ g ′ ( x ) d x = g ( ∞ ) − [ g ( ∞ ) − g ( 0 )] = g ( 0 ) = ∫ − ∞ ∞ g ( x ) δ ( x ) d x
Therefore,
d d x θ ( x − x ′ ) = δ ( x ) \frac{\mathrm{d}}{\mathrm{d}x}\theta(x-x^{\prime})=\delta(x) d x d θ ( x − x ′ ) = δ ( x )
■ ~\tag*{$\blacksquare$} ■
Exercise 1.10.4 A string is displaced as follows at t = 0 t=0 t = 0 :
ψ ( x , 0 ) = 2 x h L , 0 ⩽ x ⩽ L 2 = 2 h L ( L − x ) , L 2 ⩽ x ⩽ L \begin{aligned}
\psi(x, 0) & =\frac{2 x h}{L}, & & 0 \leqslant x \leqslant \frac{L}{2} \\
& =\frac{2 h}{L}(L-x), & & \frac{L}{2} \leqslant x \leqslant L
\end{aligned} ψ ( x , 0 ) = L 2 x h , = L 2 h ( L − x ) , 0 ⩽ x ⩽ 2 L 2 L ⩽ x ⩽ L
Show that
ψ ( x , t ) = ∑ m = 1 ∞ sin ( m π x L ) cos ω m t ⋅ ( 8 h π 2 m 2 ) sin ( π m 2 ) \psi(x, t)=\sum_{m=1}^{\infty} \sin \left(\frac{m \pi x}{L}\right) \cos \omega_m t \cdot\left(\frac{8 h}{\pi^2 m^2}\right) \sin \left(\frac{\pi m}{2}\right) ψ ( x , t ) = m = 1 ∑ ∞ sin ( L mπ x ) cos ω m t ⋅ ( π 2 m 2 8 h ) sin ( 2 πm )
Solution. We start from equation (1.10.55)
∣ ψ ( t ) ⟩ = ∑ m = 1 ∞ ∣ m ⟩ ⟨ m ∣ ψ ( 0 ) ⟩ cos ω m t , ω m = m π L |\psi(t)\rangle=\sum_{m=1}^{\infty}|m\rangle\langle m \mid \psi(0)\rangle \cos \omega_m t,\qquad \omega_m=\frac{m \pi}{L} ∣ ψ ( t )⟩ = m = 1 ∑ ∞ ∣ m ⟩ ⟨ m ∣ ψ ( 0 )⟩ cos ω m t , ω m = L mπ
Then
ψ ( x , t ) = ⟨ x ∣ ψ ( t ) ⟩ = ∑ i n = 1 ∞ ⟨ x ∣ m ⟩ ⟨ m ∣ ψ ( 0 ) ⟩ cos ω m t \psi(x, t)=\langle x \mid \psi(t)\rangle=\sum_{i n=1}^{\infty}\langle x \mid m\rangle\langle m \mid \psi(0)\rangle \cos \omega_m t ψ ( x , t ) = ⟨ x ∣ ψ ( t )⟩ = in = 1 ∑ ∞ ⟨ x ∣ m ⟩ ⟨ m ∣ ψ ( 0 )⟩ cos ω m t
From equation (1.10.55), we have
⟨ x ∣ m ⟩ = ψ m ( x ) = ( 2 L ) 1 2 sin m π x L \langle x \mid m\rangle=\psi_m(x)=\left(\frac{2}{L}\right)^{\frac{1}{2}} \sin \frac{m \pi x}{L} ⟨ x ∣ m ⟩ = ψ m ( x ) = ( L 2 ) 2 1 sin L mπ x
Therefore,
⟨ m ∣ ψ ( 0 ) ⟩ = ∫ 0 L ( 2 L ) 1 2 sin m π x L ⋅ ψ ( x , 0 ) d x = ( 2 L ) 1 2 [ ∫ 0 L 2 2 x h L sin m π x L d x + ∫ L 2 L 2 h L ( L − x ) sin m π x L d x ] \begin{aligned}
\langle m \mid \psi(0)\rangle & =\int_0^L\left(\frac{2}{L}\right)^{\frac{1}{2}} \sin \frac{m \pi x}{L} \cdot \psi(x, 0) \,\mathrm{d} x \\
& =\left(\frac{2}{L}\right)^{\frac{1}{2}}\left[\int_0^{\frac{L}{2}} \frac{2 x h}{L} \sin \frac{m \pi x}{L} \mathrm{d} x+\int_{\frac{L}{2}}^L \frac{2 h}{L}(L-x) \sin \frac{m \pi x}{L} \mathrm{d} x\right]
\end{aligned} ⟨ m ∣ ψ ( 0 )⟩ = ∫ 0 L ( L 2 ) 2 1 sin L mπ x ⋅ ψ ( x , 0 ) d x = ( L 2 ) 2 1 [ ∫ 0 2 L L 2 x h sin L mπ x d x + ∫ 2 L L L 2 h ( L − x ) sin L mπ x d x ]
where
∫ 0 L 2 2 x h L sin m π x L d x = − 2 h L ⋅ L m π ∫ 0 L 2 x d cos m π x L = − 2 h L ⋅ L m π x cos m π x L ∣ 0 L 2 + 2 h L ⋅ L m π ∫ 0 L 2 cos m π x L d x = − 2 h 2 ⋅ L 2 cos m π 2 + 2 h L ( L m π ) 2 sin m π x L ∣ 0 L 2 = − h L m π cos m π 2 + 2 h L ( L m π ) 2 sin m π 2 \begin{aligned}
\int_0^{\frac{L}{2}} \frac{2 x h}{L} \sin \frac{m \pi x}{L} \mathrm{d} x &=-\frac{2 h}{L} \cdot \frac{L}{m \pi} \int_0^{\frac{L}{2}} x \mathrm{d} \cos \frac{m \pi x}{L} \\
& =-\left.\frac{2 h}{L} \cdot \frac{L}{m \pi} x \cos \frac{m \pi x}{L}\right|_0 ^{\frac{L}{2}}+\frac{2 h}{L} \cdot \frac{L}{m \pi} \int_0^{\frac{L}{2}} \cos \frac{m \pi x}{L} \mathrm{d} x \\
& =-\frac{2 h}{2} \cdot \frac{L}{2} \cos \frac{m \pi}{2}+\left.\frac{2 h}{L}\left(\frac{L}{m \pi}\right)^2 \sin \frac{m \pi x}{L}\right|_0 ^{\frac{L}{2}} \\
& =-\frac{h L}{m \pi} \cos \frac{m \pi}{2}+\frac{2 h}{L}\left(\frac{L}{m \pi}\right)^2 \sin \frac{m \pi}{2}
\end{aligned} ∫ 0 2 L L 2 x h sin L mπ x d x = − L 2 h ⋅ mπ L ∫ 0 2 L x d cos L mπ x = − L 2 h ⋅ mπ L x cos L mπ x ∣ ∣ 0 2 L + L 2 h ⋅ mπ L ∫ 0 2 L cos L mπ x d x = − 2 2 h ⋅ 2 L cos 2 mπ + L 2 h ( mπ L ) 2 sin L mπ x ∣ ∣ 0 2 L = − mπ h L cos 2 mπ + L 2 h ( mπ L ) 2 sin 2 mπ
∫ L 2 L 2 h L ( L − x ) sin m π x L d x = ∫ L 2 L 2 h L ⋅ L sin m π x L d x − ∫ L 2 L 2 h x L sin m π x L d x = − 2 h ⋅ L m π cos m π x L ∣ L 2 L + 2 h L ⋅ L m π ∫ L 2 L x d cos m π x L = 2 h L m π cos m π 2 − 2 h L m π cos m π + 2 h L ⋅ L m π x cos m π x L ∣ L 2 L − 2 h m π ∫ L 2 L cos m π L d x = 2 h L m π cos m π 2 − 2 h L 2 m π cos m π + 2 h L 2 n π cos m π − h L m π cos m π 2 − 2 h m π ⋅ L m π sin m π x L ∣ L 2 L = h L m π cos m π 2 − 2 h L ( L m π ) 2 sin m π ⏟ = 0 + 2 h L ( L m π ) 2 sin m π 2 \begin{aligned}
&\int_{\frac{L}{2}}^L \frac{2 h}{L}(L-x) \sin \frac{m \pi x}{L} \mathrm{d} x=\int_{\frac{L}{2}}^L \frac{2 h}{L} \cdot L \sin \frac{m \pi x}{L} \mathrm{d} x-\int_{\frac{L}{2}}^L \frac{2 h x}{L} \sin \frac{m \pi x}{L} \mathrm{d} x \\
& =-\left.2 h \cdot \frac{L}{m \pi} \cos \frac{m \pi x}{L}\right|_{\frac{L}{2}} ^L+\frac{2 h}{L} \cdot \frac{L}{m \pi} \int_{\frac{L}{2}}^L x \mathrm{d} \cos \frac{m \pi x}{L} \\
& =\frac{2 h L}{m \pi} \cos \frac{m \pi}{2}-\frac{2 h L}{m \pi} \cos m \pi+\left.\frac{2 h}{L} \cdot \frac{L}{m \pi} x \cos \frac{m \pi x}{L}\right|_{\frac{L}{2}} ^L-\frac{2 h}{m \pi} \int_{\frac{L}{2}}^L \cos \frac{m \pi}{L} \mathrm{d} x \\
& =\frac{2 h L}{m \pi} \cos \frac{m \pi}{2}\cancel{-\frac{2 h L}{2 m \pi} \cos m \pi}+\cancel{\frac{2 h L}{2 n \pi} \cos m \pi}-\frac{h L}{m \pi} \cos \frac{m \pi}{2}-\left.\frac{2 h}{m \pi} \cdot \frac{L}{m \pi} \sin \frac{m \pi x}{L}\right|_{\frac{L}{2}} ^L \\
& =\frac{h L}{m \pi} \cos \frac{m \pi}{2}-\underbrace{\frac{2 h}{L}\left(\frac{L}{m \pi}\right)^2 \sin m \pi}_{=0}+\frac{2 h}{L}\left(\frac{L}{m \pi}\right)^2 \sin \frac{m \pi}{2}
\end{aligned} ∫ 2 L L L 2 h ( L − x ) sin L mπ x d x = ∫ 2 L L L 2 h ⋅ L sin L mπ x d x − ∫ 2 L L L 2 h x sin L mπ x d x = − 2 h ⋅ mπ L cos L mπ x ∣ ∣ 2 L L + L 2 h ⋅ mπ L ∫ 2 L L x d cos L mπ x = mπ 2 h L cos 2 mπ − mπ 2 h L cos mπ + L 2 h ⋅ mπ L x cos L mπ x ∣ ∣ 2 L L − mπ 2 h ∫ 2 L L cos L mπ d x = mπ 2 h L cos 2 mπ − 2 mπ 2 h L cos mπ + 2 nπ 2 h L cos mπ − mπ h L cos 2 mπ − mπ 2 h ⋅ mπ L sin L mπ x ∣ ∣ 2 L L = mπ h L cos 2 mπ − = 0 L 2 h ( mπ L ) 2 sin mπ + L 2 h ( mπ L ) 2 sin 2 mπ
Then
⟨ m ∣ ψ ( 0 ) ⟩ = ( 2 L ) 1 2 4 h L ( L m π ) 2 sin m π 2 \langle m \mid \psi(0)\rangle=\left(\frac{2}{L}\right)^{\frac{1}{2}} \frac{4h}{L}\left(\frac{L}{m\pi}\right)^{2}\sin\frac{m\pi}{2} ⟨ m ∣ ψ ( 0 )⟩ = ( L 2 ) 2 1 L 4 h ( mπ L ) 2 sin 2 mπ
Therefore,
ψ ( x , t ) = ∑ m = 1 ∞ ( 2 L ) 1 2 sin m π x L ⋅ ( 2 L ) 1 2 ⋅ 4 h L ( L m π ) 2 sin m π 2 cos ω m t = ∑ m = 1 ∞ sin ( m π x L ) cos ω m t ⋅ ( 8 h π 2 m 2 ) sin ( π m 2 ) \begin{aligned}
\psi(x, t) & =\sum_{m=1}^{\infty}\left(\frac{2}{L}\right)^{\frac{1}{2}} \sin \frac{m \pi x}{L} \cdot\left(\frac{2}{L}\right)^{\frac{1}{2}} \cdot \frac{4 h}{L}\left(\frac{L}{m \pi}\right)^2 \sin \frac{m \pi}{2} \cos \omega_{m} t \\
& =\sum_{m=1}^{\infty} \sin \left(\frac{m \pi x}{L}\right) \cos \omega_{m} t \cdot\left(\frac{8 h}{\pi^2 m^2}\right) \sin \left(\frac{\pi m}{2}\right)
\end{aligned} ψ ( x , t ) = m = 1 ∑ ∞ ( L 2 ) 2 1 sin L mπ x ⋅ ( L 2 ) 2 1 ⋅ L 4 h ( mπ L ) 2 sin 2 mπ cos ω m t = m = 1 ∑ ∞ sin ( L mπ x ) cos ω m t ⋅ ( π 2 m 2 8 h ) sin ( 2 πm )
■ ~\tag*{$\blacksquare$} ■